Sunday, February 26, 2012

Popper contra computationalism

Karl Popper was an important critic of materialist theories of the mind.  His most significant and original criticism is an argument against the possibility of a causal theory of intentionality -- an argument I discuss at length in my recent paper “Hayek, Popper, and the Causal Theory of the Mind.”  But Popper also put forward, albeit sketchily, an argument that implies the impossibility of a computational theory of the mind in particular.  The argument is presented in The Self and Its Brain, a book he co-wrote with neuroscientist John Eccles.  It foreshadows arguments later presented by John Searle and by proponents of what has come to be known as the “argument from reason,” such as Victor Reppert and William Hasker.

As I note in my recent paper (and had reason to note in an earlier post), Popper distinguishes four major functions of language.  There is, first of all, the expressive function, which involves “an outward expression of an inner state” (The Self and Its Brain, p. 58).  Here language operates in a way comparable to the sound an engine makes when it is revved up, or an animal’s cry when in pain.  The second, signaling function adds to the expressive function the generation of a reaction in others.  Popper compares it to the danger signals an animal might send out in order to alert other animals, and to the way a traffic light signals the possible presence of cars even when there are none about.  The difference between the expressive and signaling functions would seem to parallel Fred Dretske’s distinction between “natural meaning” (or meaningn) and “functional meaning” (or meaningf), which I discussed some time back in a post on Dretske.  Meaningn or “natural meaning,” it will be recalled, amounts to nothing more than an effect’s indicating the presence of its cause, as spots on the face indicate the presence of measles.  There is no possibility of misrepresentation here, since an effect will meann whatever it is that happens to cause it.  Hence if the spots on someone’s face were caused, not by measles but instead by an allergic reaction of some sort, then that, rather than measles, is what they will meann.  Popper’s “expressive function” seems more or less the same insofar as he appears to think that an effect (the sound of the engine, the animal’s cry of pain, or someone’s angry and spontaneous utterance of the appropriate expletive when stepping in something at the dog park) will “express” whatever inward state it is that happens to cause it.  The possibility of misrepresentation only clearly enters the picture with the “signaling function,” just as it does (at least if Dretske’s account succeeds) with meaningf or “functional meaning.”  An internal state or utterance might meanf that such-and-such is present even when it is not; similarly, it might in Popper’s sense “signal” the presence of something (predators, cars, or the headache your wife claims she is having) even when that something is not really there.  

Popper allows that these two elementary functions of language might be explicable in causal terms.  What he regards as inexplicable in such terms are the remaining two functions.  The descriptive function of language involves the expression of a proposition, something that can be either true or false.  The paradigm here would be the utterance of a declarative sentence, such as “Roses are red,” “Two and two make four,” or “There is a predator in the area.”  Notice that the latter example differs from an animal’s cry of warning in having a conceptual structure.  A bird’s squawk might cause another bird to feel fear and take flight.  What it does not do is convey an abstract concept like eagle, predator, or danger, and thus it does not convey the sort of propositional content that presupposes such concepts.  (Popper tentatively allows at p. 58 of The Self and its Brain that at least some animal behavior “may perhaps” involve a descriptive component and not mere signaling, giving the bee’s dance as a possible example.  I don’t find this plausible myself, but nothing in what follows rides on the issue.)  Finally, the argumentative function of language involves the expression of an inference from one or more propositions to another in a manner than can be said to be either valid or invalid, as when we reason from All men are mortal and Socrates is a man to the conclusion that Socrates is mortal.

It is Popper’s treatment of the “descriptive function” of language that indicates what he takes to be problematic about the notion of a causal theory of intentionality.  Again, I examine his argument against the possibility of such a theory in detail in “Hayek, Popper, and the Causal Theory of the Mind.”  It is in Popper’s treatment of the “argumentative function” that we find his implicit objection to computational theories of the mind.  (See The Self and Its Brain, pp. 75-81.)  Like the descriptive function, the argumentative function is something that in Popper’s view cannot be accounted for in causal terms, and he gives a separate argument to this effect.  Though he does not claim that this argument strictly refutes materialism, he says that it shows “that materialism has no right to claim that it can be supported by rational argument”; in particular, it shows that materialism, even if it were true, “is incompatible with… the acceptance of the standards of critical argument” insofar as “these standards appear from the materialist point of view as an illusion, or at least as an ideology” (p. 81).  The nerve of Popper’s argument is contained in the following passage:

The property of a brain mechanism or a computer mechanism which makes it work according to the standards of logic is not a purely physical property, although I am very ready to admit that it is in some sense connected with, or based upon, physical properties.  For two computers may physically differ as much as you like, yet they may both operate according to the same standards of logic.  And vice versa; they may differ physically as little as you may specify, yet this difference may be so amplified that the one may operate according to the standards of logic, but not the other.  This seems to show that the standards of logic are not physical properties.  (The same holds, incidentally, for practically all relevant properties of a computer qua computer.) (p. 79)

Unfortunately, while this is suggestive, Popper does not develop the argument in a formal way; the passage quoted is taken from an imagined dialogue between a “Physicalist” and an “Interactionist,” and Popper lets the responses of the latter stand in for an explicit formulation.  But the overall thrust of the argument can be reconstructed by comparison with some clearly related ideas to be found in the work of John Searle, on the one hand, and the work of proponents of what has been called the “argument from reason” on the other.

The context makes it evident that Popper intends to make both a narrow point against any attempt to explain human rationality specifically on the model of the modern digital computer, and a more general but related point against any materialist attempt to explain rationality in causal terms.  It is with respect to the former point that we find a clear parallel with Searle.  In the passage quoted, Popper says that “practically all relevant properties of a computer qua computer… are not physical properties.”  This may seem odd given that he also allows that “the property of… a computer mechanism which makes it work according to the standards of logic is… in some sense connected with, or based upon, physical properties.”  But Popper also points out that the reason a computer operates according to logical principles is that it “has been designed by us – by human minds – to work like this” (p. 76).  Its operations mirror the semantic features of linguistic symbols and their logical relationships, just as the words written in ink on a piece of paper do; but the semantics and the logical relationships are no more inherent to the physical properties in the case of the computer than they are in the case of the ink marks.  In both cases they are imparted to the physical phenomena by us – by programmers and users in the case of computers, and by writers and readers in the case of written words – rather than derived from the physical phenomena.  Hence they can hardly provide a model of how rational thought processes might be explained in purely physical terms.

Searle’s version of this line of argument emphasizes that the key notions of the modern theory of computation – “symbol manipulation,” “syntactical rules,” “information processing,” and the like – are not definable in terms of the properties attributed to material systems by physical science, but are observer-relative, existing in a physical system only insofar as some interpreting mind attributes computational properties to it.  Hence the very idea that the mind might be explained in terms of computation is incoherent.  The argument can be summarized as follows:

1. Computation involves symbol manipulation according to syntactical rules.

2. But syntax and symbols are not definable in terms of the physics of a system.

3. So computation is not intrinsic to the physics of a system, but assigned to it by an observer.

4. So the brain cannot coherently be said to be intrinsically a digital computer.

(Searle develops this argument in his paper “Is the Brain a Digital Computer?” and in chapter 9 of his book The Rediscovery of the Mind.  Note that this argument is different from Searle’s better known “Chinese Room” argument.)

There is a clear parallel between this Popper-Searle argument against a computational theory of rationality and Popper’s argument against causal theories of intentionality (which I examine in the paper linked to above).  In both cases, the materialist or physicalist is accused of making use of notions (certain causal notions in the one case, computational ones in the other) to which he is not entitled given his working assumption that the only genuine features of reality are those describable in the language of physical science.  

It is in his application of this basic idea to a critique of any possible causal account of rationality that Popper’s position resembles the anti-materialist “argument from reason.”  This is a label that has recently come to be applied to a family of related arguments to be found in the work of thinkers as diverse as Popper, J. B. S. Haldane (whom Popper cites as an influence), C. S. Lewis, Alvin Plantinga, Victor Reppert, and William Hasker.  (For a useful overview, see Reppert’s article “The Argument from Reason” in William Lane Craig and J. P. Moreland, eds., The Blackwell Companion to Natural Theology.  I discuss and defend the argument in chapter 6 of Philosophy of Mind.)  There are significant differences between these writers’ respective statements of the argument, but a “generic” version might go as follows:

1. Materialism holds that thinking consists of nothing more than the transition from one material process in the brain to another in accordance with causal laws (whether these transitions are conceived of in terms of the processing of symbols according to the rules of an algorithm à la computationalism, or on some other model).

2. Material processes have their causal efficacy, including their ability to generate other material processes, only by virtue of their physical properties (i.e. those described by physical science), and not by virtue of any meaning or semantic content that might be associated with them.  (For example, punching the symbols “1,” “+,” “1,” and “=” into a calculator will generate the further symbol “2” whether or not we associate the standard arithmetical meanings with these symbols or instead assign to them some eccentric meanings, because the electronic properties of the calculator alone are what determine what symbols get displayed.  Similarly, neural processes that are in fact associated with the thought that all men are mortal and the thought that Socrates is a man would still generate the neural process that is in fact associated with the thought that Socrates is mortal even if these neural processes had all been associated with some other meanings instead, because the neurophysiological properties of the processes alone are what determine which further processes get generated.)

3. But one thought can serve as a rational justification of another thought only by virtue of the meaning or semantic content of the thoughts.  (For example, it is only because we associate the symbols “1,” “+,” “1,” “=,” and “2” with the standard meanings that “1 + 1 = 2” expresses an arithmetical truth.  Similarly, it is only because “All men are mortal,” “Socrates is a man,” and “Socrates is mortal” have the meanings they do that the first two sentences logically entail the third, and only when the neural processes in question are associated with the corresponding thoughts that the first two provide a rational justification for believing the third.)

4. So if materialism is true, then there is nothing about our thought processes that can make one thought a rational justification of another; for their physical and causal relations alone, and not their semantic and logical relations, determine which thought follows which.

5. So if materialism is true, none of our thoughts ever is rationally justified.

6. But this includes the thoughts of materialists themselves.

7. So if materialism is true, then it cannot be rationally justified; the theory undermines itself.

The upshot of this argument is that instantiating causal relations, of whatever sort, does not by itself amount to instantiating logical relations; and this is precisely what Popper is getting at in the passage above when he says that “brain mechanisms” or “computer mechanisms” may “differ physically as little as you may specify, yet this difference may be so amplified that the one may operate according to the standards of logic, but not the other.”  Hence even if we concede that certain causal processes are necessary conditions for our reasoning logically (which Popper allows insofar as he says that our ability to follow standards of logic is “in some sense connected with, or based upon, physical properties”), they are not sufficient conditions – in which case there can be no (purely) causal explanation of our ability to reason logically.

Step 2 of the argument seems to follow from the standard materialist assumption that whatever happens in the natural world supervenes on what happens at the microphysical level of nature – the level of the basic particles described by physics and the laws governing them – together with the further materialist assumption that meaning or semantic content is not a microphysical property, whatever else the materialist wants to say about it.  That this appears to make the meanings of our thoughts “epiphenomenal” or causally irrelevant to what happens in the world is known as “the problem of mental causation.”  Of course, the meanings of our thoughts seem to have an effect on what we say and do; in particular, it certainly seems to us that we judge an inference like All men are mortal and Socrates is a man, therefore Socrates is mortal to be rational because of the meanings associated with these words, and would not judge it to be rational if they had some different content.  But Popper’s point is that, if materialism is true, then we can have no grounds for believing that what seems to be the case really is the case.  Perhaps the inference in question is in fact irrational, while an inference that seems irrational to us, like All men are mortal, and Grandma drives a Buick, therefore robots are stealing my luggage is a paradigm of rational thinking.  Perhaps we don’t see this for the same reason the calculator would spit back “2” in response to the sequence “1 +1 =” even if the latter set of symbols expressed the question Does Grandma drive a Buick? and the former expressed the bizarre answer No, robots are stealing my luggage – namely for the reason that only the physical properties of events occurring in both calculators and brains, and not any semantic or logical properties associated with them, determine what effects they will generate.

For this reason Popper claims that materialism tends to reduce the argumentative function of language no less than the descriptive function to the sub-rational expressive and signaling functions, and thereby tends also to “make us blind to the difference between propaganda, verbal intimidation, and rational argument” (The Self and Its Brain, p. 59).  Now Popper presumably thought that his friend F. A. Hayek’s account of the mind was open to this sort of criticism, just as it was open to the criticism I discuss in “Hayek, Popper, and the Causal Theory of the Mind.”  There is irony in this, for Hayek himself accuses others of what Popper implicitly accuses him.  Hayek argued in The Counter-Revolution of Science, an important critique of scientism, that “the ground for a thorough irrationalism” lay implicit in any view of human beings aimed at “uncovering hidden causes which, unknown to the thinker, have determined his conclusions.” (p. 159).  His target was the relativist idea that a person’s race or class situation determines what he thinks.  Popper’s claim is that the materialist view that our thoughts are determined by the hidden causal processes uncovered by physical science is no less implicitly irrationalist.

265 comments:

  1. Quite happy to see two posts in a row on a topic that's (sadly) overlooked.

    It was also fun reading your paper on Popper v. Hayek, and your reconstruction of Popper's argument was similar to my own understanding of it.

    Best,
    d

    ReplyDelete
  2. Feser,

    Upon reading the rest of the post, there's one point where I think your reductio could be stronger: if materialism and the possibility justification are in conflict, the materialist can reject justification as Popper did, but for different reasons.

    However, if the reductio can be reformulated as a conflict between materialism and the possibility of criticism, then the materialist does not have that route available.

    Rather, they'd have to deny that there can be such thing as a critical argument, which is quite a thing to deny, no?

    ReplyDelete
  3. "A bird’s squawk might cause another bird to feel fear and take flight. What it does not do is convey an abstract concept like eagle, predator, or danger, and thus it does not convey the sort of propositional content that presupposes such concepts. "

    You say a sentence or two later that maybe nothing rides on this--if so, good, because from what I've read there've been studies of monkey warning cries where it turns out that there are different cries for different sorts of threats. One for eagle and one for ground predator, for instance. I don't know if you'd call that propositional content, or if it matters at all for your argument whether some animals might be able to express some sort of content. (The chimps who can use sign language may not have all the elements of human language, but they are expressing some sort of propositional content with their signing.)

    Though again, this might be irrelevant. I just stopped at this point in your piece to object.

    Donald

    ReplyDelete
  4. "But one thought can serve as a rational justification of another thought only by virtue of the meaning or semantic content of the thoughts."

    This is an unfounded assumption. The use of the English term "rational justification," rather than some mythical language-independent term for the same concept, presupposes a specific language. Rational justification of propositions in English can, in fact, be checked by machine (although usually most theorem provers use a more specialized language -- albeit still one recognizably based on English.)

    d -- if you're going to choose definitions in such a way as to deny the physical existence of things like "critical arguments," which can be found in such physical artifacts as ink on paper, the problem is with your definitions, not with Physicalism.

    Honestly, I find these sorts of arguments have all the sophistication of that old Chestnut "why are you hitting yourself."

    ReplyDelete
  5. Okay, I finished, and yes, my objection wasn't relevant at all. It only matters if we got into an argument about the extent to which animals are capable of rational thought.

    An actually relevant objection though--I find the argument from reason somewhat compelling, but I'm not sure why a naturalist couldn't simply argue that for some species (those which aren't hardwired to behave in the stereotypical ways that we call "instinct") natural selection produced material objects called brains that were capable of reaching valid conclusions and this was a useful survival trait. Yes, it's true we can never really know if we are in fact reasoning correctly, but it is at least a self-consistent story--that is, it is logically possible that this is what happened and while we don't and can't even in principle know for sure, it might have happened that way and we might as well act on the belief that it did. (Maybe a naturalist would give a more forceful argument than the one I just gave.)

    Donald

    ReplyDelete
  6. Donald,

    Could you explain how meaning is inherent in specific patterns of ink on paper and not others?

    ReplyDelete
  7. I'm the second anonymous, not Donald, but I suspect your criticism is directed at me.

    "Could you explain how meaning is inherent in specific patterns of ink on paper and not others?"

    To deny this is to claim that you could produce the same pattern of ink on paper in the same situation, and it would mean something different. So, analogously, if you insist that meaning is not inherent in the physical objects to which we attribute meaning, why don't you post the same characters in the same order you did last time, but this time without the meaning?

    I will also note that my argument does not rely on all possible senses of the word "meaning" existing. "meaning" like most natural language terms is mildly ambiguous. In some contexts it could mean "correct English translation as judged by a native speaker" or something of the sort -- this is not problematic. The problems only arise when you assume that meaning must both be abstract and expressible without resorting to language.

    ReplyDelete
  8. Hello d,

    Interesting point. Yes, I think the argument could be stated that way as well. Whether we're considering justification or criticism, we have to be able to reason according to valid forms like modus ponens. And that's what Popper says cannot be reduced to causal processes.

    Anonymous at 1:56,

    This is an unfounded assumption.

    Is it, now. Please explain, then, why:

    All men are mortal.
    Socrates is a man.
    Therefore, Socrates is mortal.

    is valid, without reference to the meanings of any of the terms involved. (Note that it will not do to say that the argument is valid because of its form rather than because of what the major, minor, and middle terms happen to be. For we still need to know that there is no equivocation on any of the terms, we need to know the meanings of the quantifier "All" and the copulae "is" and "are," and we need to know that these marks are supposed to be sentences in the first place.)

    To note that machines can check for validity is quite risible given what was said in the original post. Did you bother to read it? As Popper and Searle point out, machines can "check" for validity only because their designers and users have imparted such-and-such a significance to their processes, counting them as symbols, algorithms, etc. Apart from this, nothing they do counts as "checking" for validity or as computation of any kind. If you disagree with Popper and Searle, you've given no reason for doing so, but have simply begged the question.

    ReplyDelete
  9. The problems only arise when you assume that meaning must both be abstract and expressible without resorting to language.

    Nobody is saying that it must be expressible apart from language. The claim is that it isn't reducible to bits of language considered merely in terms of their physical properties. A Frege pointed out, a proposition (like the proposition that snow is white) cannot possibly be identified with any sentence or set of sentences ("Snow is white," "Schnee ist weiss," etc.) because the proposition would remain true whether or not any sentence or indeed any language at all ever existed. All the same, we can only ever express the proposition via some sentence or other.

    ReplyDelete
  10. Ugghh!!!
    Could someone tell me what intentionality is?


    Sorry!

    ReplyDelete
  11. Intentionality is the power of minds to be about, to represent, or to stand for, things, properties and states of affairs. Not to be confused with intensionality.

    ReplyDelete
  12. Ed

    "This is an unfounded assumption.

    Is it, now. Please explain, then, why: ..."

    You're basically asking me to write a natural language processor in a blog post -- which looks for all the world like a hard, but ultimately tractable, AI problem. In any event "please demonstrate that my assumption is false." is not a sufficient response to the claim that you have made an unfounded assumption. You need to demonstrate how your assumption follows deductively from premises we share.

    not Donald

    ReplyDelete
  13. Not Donald,

    Your assertions presuppose that the issue at hand is a matter of "writing a natural language processor," an "AI problem," etc. -- which is why you are begging the question, since whether this is so is, of course, precisely what Popper, Searle, and I deny.

    BTW, that "all the world" is begging a certain question doesn't make it any less a begged question.

    ReplyDelete
  14. "A Frege pointed out, a proposition (like the proposition that snow is white) cannot possibly be identified with any sentence or set of sentences ('Snow is white', 'Schnee ist weiss', etc.) because the proposition would remain true whether or not any sentence or indeed any language at all ever existed."

    I'm really at a loss with regard to what this is supposed to prove. It seems perfectly consistent to regard the proposition denoted by "snow is white" as a sort of a fiction. I mean we can make all kinds of true counter-factual claims about Unicorns, and they definitely don't exist. In any event there's at least one sense of existence for which the proposition denoted by the sentence clearly doesn't exist (the physical sense.) So, whether we say the proposition exists or not seems as much a matter of semantics as metaphysics.

    -not donald

    ReplyDelete
  15. "Your assertions presuppose that the issue at hand is a matter of 'writing a natural language processor,' an 'AI problem,' "

    Well. Specifically the problem is to program a computer to respond to the yes/no question "does A rationally justify B?" where A and B are finite strings of English prose, exactly as a native English speaker would.

    If you don't mind a non-constructive proof that this is possible, you could simply note that the maximum amount of English prose a human can process in a lifetime is limited to less than a terabyte of unicode. So you could just do the whole thing as a table look-up, assuming there's always a right answer. -- you can't actually do it that way in practice, but this at least demonstrates that any demonstration of the impossibility of natural language processing must assume something about physical limitations on the amount of computation that can practically be done in the known universe. Needless to say I don't see anything that even hints of this in your metaphysical argument.

    Oh. And as much as you accuse me of begging the question, you STILL have not provided a foundation for your unfounded assumption.

    --not donald

    ReplyDelete
  16. Searle’s version of this line of argument emphasizes that the key notions of the modern theory of computation – “symbol manipulation,” “syntactical rules,” “information processing,” and the like – are not definable in terms of the properties attributed to material systems by physical science, but are observer-relative, existing in a physical system only insofar as some interpreting mind attributes computational properties to it.

    Searle is very confused. Unpacking his confusion is not all that simple, because his critque is somewhat valid when applied to a large class of AI programs, particularly those that date from around the time he floated this line of attack. But that does not mean they apply to all computational systems of all time. I think Dennett and Hofstadter did a pretty thorough job on Searle's confusion a few decades ago in The Mind's I.

    Let me point to something I said here awhile back, when I introduced the example of a ribosome, which is a physical symbol manipulation system, that obeys syntactic and semantic rules to intepret symbols (RNA codons) as instructions for synthesizing proteins. Now obviously a ribosome is not a mind. That's good, because people seem to get very confused when thinking about minds. But hopefully this example of a physical symbol manpulation system can serve to illustrate that such things are possible without and outside observing mind.


    BTW, Searle (in his Chinese Room paper) says this: " "Could a machine think?" My own view is that only a machine could think, and indeed only very special kinds of machines, namely brains and mchines that had the same causal powers as brains." And follows that with some rather unenlightening discussion about the difference between programs and machines.

    So I don't know where that puts him relative to Popper, or you. It is true that programs by themselves don't have causal powers -- strictly speaking, they are just inert strings of symbols. But when they are executed on physical computers, lo and behold, the combination of machine and program does have causal powers.

    ReplyDelete
  17. Saying Searle is confused and then referencing Dennett's treatment of Searle as clearing up the confusion?


    wow

    ReplyDelete
  18. goddinpotty you shouldn't have linked that because it really is damning for you. You look pretty bad in the ensuing conversation on mechanism and telos.

    Maybe you're just that humble.

    ReplyDelete
  19. "That's good, because people seem to get very confused when thinking about minds. But hopefully this example of a physical symbol manpulation system can serve to illustrate that such things are possible without and outside observing mind."

    Let me share a quote with you.

    In any event, the contemporary biologist's absolute favorite place to relocate teleology while pretending it doesn't exist is not the human mind (pace Searle) and it is not Dennett's "Mother Nature" either; it is DNA. If there is one thing biologists gush over as much as natural selection, it is the discovery of this famous molecule and its structure - and for good reason, for it was a very great discovery indeed. And why was it so important? Because, we are constantly and rightly told, DNA contains the "information," "code," "instructions," "data," or "blueprint" required to build an organism; occasionally, given the current fad for computer jargon, one hears of "software," "programming," and the like too. Such language absolutely permeates biologists' descriptions of the function of DNA, and there is no way accurately to convey what DNA does without something like it. Notice, though, that every single one of these concepts, and others frequently used to describe the nature of DNA, smacks of the sort of intentionality or meaningfulness characteristic of human minds. [...] So, what modern biology reveals to us is the existence of a physical structure that "points to" or "aims at" something beyond itself and yet is entirely unconscious. Where have we heard that before? Why, in Aristotle of course.

    Do you know what I'm quoting from?

    ReplyDelete
  20. Of course, the meanings of our thoughts seem to have an effect on what we say and do; in particular, it certainly seems to us that we judge an inference like All men are mortal and Socrates is a man, therefore Socrates is mortal to be rational because of the meanings associated with these words, and would not judge it to be rational if they had some different content.

    Yep, that's how it seems; but that doesn't tell us what's really going on underneath. When I write a program and then run it, it seems like the semantics of the program are being played out on the screen - and, indeed, there's a sense in which they are. But there's also a sense in which the program, when it's running, has no meaningful semantics: when it's running, it's just a bunch of electrical currents and potentials on a circuit board, and is wholly physically instantiated. The semantics of the programming language are an abstraction we use to reason about and describe the way in which we want the physical computer to behave, but the actual behavior doesn't have any semantic properties.

    Anyway, here's the problem I have taking non-materialists seriously:
    You want to say that there are problems with the materialist account of meaning and intentionality; OK, fine, there's certainly a lot of stuff to work out there. But there seems to be a rather massive beam in your own eye: if meaning and intentionality are non-material/metaphysical properties of minds, how do they cause and constrain the physical behavior of our bodies? When your non-material mind rationally determines that you want to utilize the argumentative function of language by expressing a sentence using the keyboard of your computer, how does it make your hands do that? Or, alternatively, how does it prevent your brain and your hands from typing any old sentence, unconstrained by the non-material rationality they cannot possess? Does any non-materialist make an effort to explain this relationship between the non-material/metaphysical and the material/physical realms?

    ReplyDelete
  21. not Donald, your 6:48 response is somewhat odd.

    You suggest that there is no real problem here, because we can just claim that propositions are a kind of "fiction". This seems to concede the very point at issue. The whole argument is that computationalism is insufficient for semantic content because there are no physical properties which can explain the phenomenon of propositional equivalence.

    Your whole "rebuttal" is to argue that computationalism can explain semantic contents because semantic contents don't really exist except as a kind of useful fiction.

    But that isn't an explanation; it is an elimination and thus it doesn't help your case at all to propose it.

    Hence, you seem to agree with the following conjunction: Either semantic contents are real and physicalism is false, or physicalism is true and semantic contents are fictitious.

    ReplyDelete
  22. @DDT -- it's pretty obviously Feser, the giveaway is the pulling Aristotle out of a hat with a triumphant flourish, as if that is supposed to prove something,

    Might be better to present an argument rather than play guessing games.

    ReplyDelete
  23. @goddinpotty:

    "Searle is very confused. Unpacking his confusion is not all that simple, because his critque is somewhat valid when applied to a large class of AI programs, particularly those that date from around the time he floated this line of attack. But that does not mean they apply to all computational systems of all time."

    Chuckle.

    Formulate Searle's objection in terms of idealized Turing machines. There, problem solved. Unless you have some reason to believe the Church-Turing thesis is false and there is a "computational system" that cannot be modeled by a Turing machine.

    @Steve Ruble:

    "But there seems to be a rather massive beam in your own eye: if meaning and intentionality are non-material/metaphysical properties of minds, how do they cause and constrain the physical behavior of our bodies? When your non-material mind rationally determines that you want to utilize the argumentative function of language by expressing a sentence using the keyboard of your computer, how does it make your hands do that? Or, alternatively, how does it prevent your brain and your hands from typing any old sentence, unconstrained by the non-material rationality they cannot possess? Does any non-materialist make an effort to explain this relationship between the non-material/metaphysical and the material/physical realms?"

    First, strictly speaking, saying that "intentionality" is a "non-material/metaphysical properties of minds" glosses over the fact that intentionality permeates all the levels reality in the form of final causes. Second, by your description, you seem to think Prof. Feser defends Cartesian dualism; well, he does not, he is a Hylemorphist dualist. The relation between the body and the soul, which is the Form of the body, is one of *formal* causation not of efficient causation, just like the relation of the Intellect to the Will is one of final causation (although the Will can efficiently cause the Intellect to say, focus its attention on a different object). There is "no beam" to be removed, it is just that the explanation will not be in the terms you are accustomed to. In other words, learn what you are criticizing and then lodge your complaints. And what the heck is "non-material rationality"?

    ReplyDelete
  24. The brain is comprised of 100 billion neurons and 100 trillion synaptic connections.

    I think intuition and metaphysics regarding the brain is about as useful as it was with quantum mechanics.

    ReplyDelete
  25. Untentured

    yeah, I agree it looks like I contradicted myself. This thing all goes back to the existence of abstract objects, which I argue is a matter of semantics.

    Thus, when I say "Of course critical arguments exist, I just printed one on the page" I am clearly using a definition of "critical argument" which does not assume that a critical argument is an abstract object. When I say, "there exists a better argument I could have given," now I am assuming that argument is an abstract object, but I also mean "exists" in a different sense -- the mathematical sense. The first statement remains true, whether or not I agree that the second statement is literally true (i.e. whether I think mathematical existence counts as "real" existence) or simply a convenient metaphor.

    existence may be defined to include both mathematical existence and physical existence or just physical existence. As long as the distinction is well defined (which I believe the Aristotelian is committed to) then things like propositions may be said to exist or not without contradiction.

    So "a human can make rationally justified statements" or even "thoughts A and B are a rational justification for thought C" may or may not entail the existence of the abstract object "rational justification" depending on the definition of existence. But changing the map does not change the territory (except insofar as the map is part of the territory.) Thus your definition of the word "existence" in no way changes how my brain works or that I can, and often do, make rationally justified statements.

    In any event my actions are caused by my beliefs, not the propositions they represent (and by the way, my unicorn example does in fact show that the existence of a belief does not entail the existence of the thing it represents.) Beliefs are not abstract objects. Physical changes in the world can make them disappear.

    -not donald

    ReplyDelete
  26. DDT

    Such language absolutely permeates biologists' descriptions of the function of DNA, and there is no way accurately to convey what DNA does without something like it.

    This is unfounded and most likely false -- although it's hard to say with the weasel words "something like it" in there, not to mention the use of "accurately" to set up a "no True Scotsman" defense.

    grodriguez

    You can say Feser is a Hylomorphic dualist and not a Cartesian dualist. The problem is that Hylomorphic dualism doesn't really do any of the work he wants it to do. Sure we can say a living human has a form -- so does a rock if we actually believe Hylomorphism. So Feser's explanation of how we're different from rocks would be the form of a human is called a soul, the form of a rock is not. Not very informative.

    In any event he thinks he can get from statements like "propositions are the formal causes of thoughts," where it's unclear how any of this is supposed to relate to anything concrecte, and where slight changes of definition make these supposed "metaphysical truths" vanish like a puff of wind, to very concrete statements like "he was crucified for us under Pontius Pilate, and suffered, and was buried, and the third day he rose again, according to the Scriptures, and ascended into heaven, and sitteth on the right hand of the Father."


    Also:
    Untentured

    To be clear, when I said to you "it looks like I contradicted myself", I was not saying I actually contradicted myself, merely that I spoke somewhat unclearly.

    The long and short of it is the existence of things like rationally justified arguments in no way requires the existence of "semantic content" unless the phrase "semantic content exists" is either defined to be tautologically true or logically equivalent to a statement about the physical world.

    -not donald

    ReplyDelete
  27. grodrigues - Yeah, I'm not convinced about this whole 'hylemorphism' thing yet. I'm working through "The Last Superstition", and I'm up around the point where that idea is introduced. But I've been seeing a couple of issues.

    When introducing Platonism/realism, he makes the point about things like "2+2=4" existing, being true, even if all minds capable of recognizing it went extinct. Indeed, even no minds capable of recognizing it ever arose. But with hylemorphism, the 'form' of a table doesn't exist without the matter instantiating it. What of the 'form' of mathematical truth, then? What 'instantiates' that, in hylemorphic terms? (I haven't read further, and I could be entirely wrong, but I have a hunch the 'divine intellect' will be invoked?)

    The 'act'/'potency' bit's somewhat iffy, too. Nothing changes unless acted upon by something else, allegedly. Except QM seems to offer counterexamples - e.g. nuclear decay. And isn't 'acting without being acted upon' practically the definition of free will?

    Even the idea of mathematical truths 'existing' is suspect. For example, he talks about how triangles have to have 180 degrees and so forth. But hyperbolic and elliptical geometries are consistent, as well - and in the real world Euclidean geometry's actually the 'special case'.

    And then there's imaginary numbers - that seems to be a mathematical truth without a physical instantiation at all (however useful electrical engineers find the concept).

    Perhaps Feser will address this stuff as I go along, but I did a check of the index and skimmed the mentions of 'free will' and that notion doesn't seem to have come up. Well, I'll keep plugging as I get time.

    ReplyDelete
  28. You can say Feser is a Hylomorphic dualist and not a Cartesian dualist. The problem is that Hylomorphic dualism doesn't really do any of the work he wants it to do. Sure we can say a living human has a form -- so does a rock if we actually believe Hylomorphism. So Feser's explanation of how we're different from rocks would be the form of a human is called a soul, the form of a rock is not. Not very informative.

    Once upon a time, there was a man who loved to argue. One day, he went to the barn, picked up a bunch of straw, and assembled a human figure with it. After painting a smilie face on its head and hanging a crucifix from its neck, he proceded to play ping-pong with it. He won the match and went back to his underground residence, where he continued the writing of his dissertation titled "On the Sweetness of the Aroma and the Delightful Contrabassoon-like Sound of My Own Farts." ~el fin~

    ReplyDelete
  29. @not donald:

    "This thing all goes back to the existence of abstract objects, which I argue is a matter of semantics."

    Wrong on both counts.

    Untenured's response is spot-on correct and it is orthogonal to whatever ontological commitments you have as far as abstract objects. Saying that (mathematical) proof can be formalized as a mathematical, finite combinatorial object, say as a tree or list of statements in a given formal language satisfying certain properties, and thus that the problem of checking the validity of a proof is well-posed with an algorithmic solution and proof verifiers can be constructed, is no response and misses the whole point of Popper's argument.

    ReplyDelete
  30. @Ray Ingles:

    "Nothing changes unless acted upon by something else, allegedly. Except QM seems to offer counterexamples - e.g. nuclear decay. And isn't 'acting without being acted upon' practically the definition of free will?"

    The first and the last sentence are incorrect. The second most probably is.

    "Even the idea of mathematical truths 'existing' is suspect. For example, he talks about how triangles have to have 180 degrees and so forth. But hyperbolic and elliptical geometries are consistent, as well - and in the real world Euclidean geometry's actually the 'special case'."

    Yawn. The existence of non-euclidean geometries is irrelevant to the truth or not of Platonism or any other form of mathematical realism. And about your last sentence, shrug shoulders. Surveying physical theories, from classical mechanics to more or less radical proposals, we find *all* sorts of geometries being employed: Symplectic geometry, conformal geometry, non-commutative geometry, infinite-dimensional spaces of all sorts (Hilbert spaces, Von-Neumann algebras, etc.), pointless spaces, cohesive infinity-topoi, etc.

    "And then there's imaginary numbers - that seems to be a mathematical truth without a physical instantiation at all (however useful electrical engineers find the concept)."

    Which kinda reinforces Prof. Feser's point...

    ReplyDelete
  31. el fin and grodrigues

    I see lots of snark but no actual arguments. Experience tells me this is as close to victory as one can get with Feserites.

    -not donald

    ReplyDelete
  32. @not donald:

    "I see lots of snark but no actual arguments. Experience tells me this is as close to victory as one can get with Feserites."

    The irony, the irony.

    ReplyDelete
  33. not Donald,

    I'm really not sure what your response is supposed to be, and I don't see how it helps to bring in semantics or abstract objects. The issue here is whether certain obvious data, like the contentfulness of thoughts, can be cashed out in physicalist terms. Saying that contentful thoughts are a "fiction" does nothing to address the basic question, nor does punting the basic question into realm of "semantics."

    If I say that point masses are merely a "useful fiction", then this is tantamount to denying that they exist. The same thing is true of propositional attitudes pace Dennett's attempts to muddy up the water using some misplaced analogies with instrumentalism e.g. "the lost sock center", the "intentional stance" et al.

    Thus, this is not a "semantic" issue, it is an ontological issue. If the semantic properties of mental states don't really exist, then I can't explain anything by appealing to them. Not any more than I can explain the trajectory of a falling body by citing the causal powers of a fictional "point mass." If it ain't real, it ain't going to cause anything.

    Thus, if you really think that semantic properties are fictional, then you are in implicitly denying that they can explain thought or behavior. This effectively committs you to either eliminativism or epiphenomenalism.

    If you really want to bite those bullets, chomp away. But then the initial point about the explanatory inadequacy of physicalism stands unchallenged.

    ReplyDelete
  34. Blogger grodrigues said...

    @not donald:

    "I see lots of snark but no actual arguments. Experience tells me this is as close to victory as one can get with Feserites."

    The irony, the irony.

    February 27, 2012 9:05 AM"


    That bears repeating. So I did.

    ReplyDelete
  35. Untenured

    If the semantic properties of mental states don't really exist, then I can't explain anything by appealing to them.

    This sentence is the crux of the problem

    For one thing, it might not be true -- does "unicorns are typically portrayed as having healing powers" serve as an explanation for some medieval narwhal hunts? Does it appeal to unicorns? It certainly looks like it does. It doesn't appeal to the existence of unicorns, but that might be a different thing than appealing to unicorns. Maybe you want to define appealing to X as making a statement from which the statement "X exists" can be rationally deduced, but in order for this to be a good definition, you have to come up with well defined rules of deduction for natural language, and in any event the statement starts looking more and more circular. Further, my original point was that changing the rules of justification within natural language so "X is rationally justified by Y" does or doesn't imply "the abstract object rational justification exists" leaves both mathematics and the natural sciences intact. So if that holds, your hypothetical definition of "appeals to" starts looking pretty ambiguous.

    I suppose I could just ask you what these words are supposed to mean straight up, since I'm not sure I know:

    what is a "semantic property of a mental state" How, given a mental state, would I personally be able to figure which semantic properties it had? The epistemological question is important here. I can't explain something in terms of things I don't know about any more than I can explain something by asserting the existence of nonexistent things.

    What exactly do you mean by explanation anyway? Usually it's an intersubjective thing. No? I explain to you why I said such and such. Doesn't the epistemological requirement become an intersubjective one. How do you come to an agreement on the meaning of words without defining things in terms of the physical world?

    And again what, above and beyond asserting that something exists, what does it mean to appeal to it?
    Does the statement "snow is white" appeal to the proposition expressed by the sentence "snow is white"?

    Anyway, thank you for providing an argument this time, although I'm afraid I find it fairly unconvincing, because it seems too semantically slippery. (by which I mean that I see no way to come to an agreement on which statements logically follow from the above quoted assumption, and absent that, I don't know what I'm denying by disagreeing with it.)

    -not donald

    ReplyDelete
  36. grodrigues,

    "Second, by your description, you seem to think Prof. Feser defends Cartesian dualism; well, he does not, he is a Hylemorphist dualist. The relation between the body and the soul, which is the Form of the body, is one of *formal* causation not of efficient causation..."

    Whatever. Here's the point: the movements of my fingers, as I type this sentence, are efficiently caused by the firing of nerves which run from my fingers to my brain. The firings of those nerves are efficiently caused by other nerves within my brain. I think it's quite plausible that all the nerves in my body work in accordance with the same physical laws we see everywhere else; that none of the atoms and molecules and electrical potentials involved do things which would be surprising to a physicist. If that is the case, then what can it mean to claim that there is some non-material thing which is really responsible for my apparent use of the "argumentative function of language"? If all the behaviors of all the components of my body are in accordance with the laws which appear to hold for all behaviors of all components of all other things in the world, what is the "soul" actually doing?

    If your god yanked all the "souls" out of all the people in the world, except for you and Edward Feser, so that all other humans were no more ensouled than the average baboon, how would you be able to tell?

    ReplyDelete
  37. "If your god yanked all the "souls" out of all the people in the world, except for you and Edward Feser, so that all other humans were no more ensouled than the average baboon, how would you be able to tell?"

    The distinct lack of bodies?

    Read up on the difference between Cartesian dualism and hylemorphic dualism, as has been suggested. Most of us here know these differences, so when you say nonsense like the above it just exposes your ignorance.

    Also, if your belief is that one should only believe in things that "won't surprise physicists", congratulations on having a belief that is practically guaranteed to not be true. Physics is not complete, and physicists reasonably expect to be surprised (again) in the future.

    ReplyDelete
  38. Anonymous,

    Read "soul" as "rational soul" if you like; since this whole post is about rationality and theories of mind, not about forms and all that, I guess didn't think it was necessary to distinguish the two.

    "Also, if your belief is that one should only believe in things that 'won't surprise physicists', congratulations on having a belief that is practically guaranteed to not be true. Physics is not complete, and physicists reasonably expect to be surprised (again) in the future."

    I assure you that physicists are not expecting to find new physics inside of humans. That's why they needed to build the LHC. See also here.

    ReplyDelete
  39. @grodriguez --

    - the Church/Turing thesis is not relevant to the discussion

    - even if it was, it only applies to idealized programs. My point was that programs running on actual computers are physical systems, and thus have plenty of causal powers. And insofar as they interact with the world, they are not programs in the classical (Turing) sense of computable functions with a single inout and output.

    ReplyDelete
  40. "Read "soul" as "rational soul" if you like; since this whole post is about rationality and theories of mind, not about forms and all that, I guess didn't think it was necessary to distinguish the two. "

    Again, if you think the soul / mind (as far as hylemorphism is concerned) has nothing to do with forms, you're just exposing your ignorance of the subject. You do so further by supposing that this is a question which can be settled one way or the other by empirical science: metaphysical views (whether naturalism, materialism, hylemorphism, panpsychism or otherwise) don't get decided in the laboratory.

    "I assure you that physicists are not expecting to find new physics inside of humans. That's why they needed to build the LHC. See also here."

    Oh boy, Carroll bloviating about things he knows nothing about. Talk about ammunition! "Well, sure, we're utterly in the dark about the mind and various other things completely outside my field of expertise, but I'm sure that nothing special will be required to fill in the blanks."

    Color me unimpressed. Especially since Carroll's line could have been repeated at various other times throughout history where it was claimed that our theory Works Great As Is and no amendments to it are required, sure there are some small lingering issues (gravity, microscopic phenomena, etc) but golly I bet we'll be able to absorb those things within the reigning paradigm (contact mechanics, classical physics).

    ReplyDelete
  41. @Steve Ruble:

    "Second, by your description, you seem to think Prof. Feser defends Cartesian dualism; well, he does not, he is a Hylemorphist dualist. The relation between the body and the soul, which is the Form of the body, is one of *formal* causation not of efficient causation...

    Whatever."

    Exactly. You do not know what you are criticizing; that was my only point. As far as your "argument" goes, it reduces to an argument from incredulity. But since by your own admission you do not even know the supporting arguments of the opposing side, your incredulity is just another name for ignorance.

    @goddinpotty:

    "- the Church/Turing thesis is not relevant to the discussion

    - even if it was, it only applies to idealized programs. My point was that programs running on actual computers are physical systems, and thus have plenty of causal powers. And insofar as they interact with the world, they are not programs in the classical (Turing) sense of computable functions with a single inout and output."

    Oh brother... All computational systems *are* physical implementations of Turing Machines. Modern computers are closer to what is called a register machine, another model for computation, and likewise equivalent to Turing machines, but as far as their intrinsic workings or what they can compute it is all the same. You said and I quote:

    "Searle is very confused. Unpacking his confusion is not all that simple, because his critque is somewhat valid when applied to a large class of AI programs, particularly those that date from around the time he floated this line of attack. But that does not mean they apply to all computational systems of all time."

    Turing machines are a model for computational systems. All known models for computation are equivalent. A computational system, whatever it computes, is a physical implementation of some Turing Machine. If there is a computational system that is not modeled by a Turing machine, the Church-Turing thesis is wrong. Searle's Chinese experiment room can be easily changed to make the the poor inhabitant of the closed cubicle to simulate a universal Turing machine (with the usual caveats, unbounded memory and time), ergo what you are saying implies that you have a counter-example to the Church-Turing thesis.

    If there is confusion is not in Searle.

    ReplyDelete
  42. Anonymous (seriously, is it that hard to make up a name?),

    I'm rather amused that your response to Carroll's post - in which he points out several different ways people have failed to understand his point in an earlier post about the same topic - is to make exactly the errors he specifically points out. Maybe you should go read it again.

    Anyway, I now understand that you think it's important to distinguish between animal, vegetable, and rational souls even when it's entirely obvious that we're talking about that thing - whatever you want to call it - which Feser thinks makes us different from animals, and that there's exactly one kind of soul I could be intending to refer to. Great.

    So: do you think that there is some thing other than the well understood physical principles which govern the behavior of atoms and molecules (at everyday energies) at work in the human body? If so, what is that thing (and how did you discover it)? If not - if everything which happens inside each human is explicable in terms of the same laws which apply outside of humans - then what explanatory power can souls/forms have with regards to anything we do?

    @grodrigues,

    Nope, you've got it wrong. I know what I'm criticizing; I just don't find it remotely plausible, nor particularly interesting. Why would anyone deeply care about theories which cannot - in principle, according to good old Anonymous - be distinguished from make believe?

    I mean, I understand that it's fun to play with semantic systems and to just see what you can work out from a set of axioms or concepts - I'm a programmer, I love doing that stuff - but there's a huge difference between doing that and actually trying to figure out what's going on in the real world. Ya'll seem to think that your word games should be able to constrain what can be truthfully said about the world, and that your word games are safe from dis-confirmation by anything in the real world. I don't think you can have it both ways.

    ReplyDelete
  43. "Searle’s version of this line of argument emphasizes that the key notions of the modern theory of computation – “symbol manipulation,” “syntactical rules,” “information processing,” and the like – are not definable in terms of the properties attributed to material systems by physical science, but are observer-relative, existing in a physical system only insofar as some interpreting mind attributes computational properties to it."

    Something's confused here, but I don't think it's Searle. What does that last clause mean? How can a property "exist in a physical system" because "some interpreting mind attributes" that property to that system? I'm pretty sure that I can't make an object have properties just by attributing those properties to the object; surely the more natural thing to say is that physical systems are the way that they are, and observers may use different terms or "notions" to describe the way that those systems are. "Insofar", in this case, must not go very far at all.

    Of course, the same thing applies to minds and the brains which run them: there are any number of words we can use to describe and try to capture or understand the things that brains do, but there's no real reason to think that those words actually point to anything other than the observable behaviors of brains.

    ReplyDelete
  44. @grodriguez -- the Church-Turing thesis is about the equivalence of certain classes of computational problem. It says exactly nothing about the mind. Well, it does say that if the mind is an algorithm, then that algorithm is realizable by a Turing machine. I fail to see the relevance to the discussion. If the mind is an algorithm, then it has the properties of an algorithm, and if not, not.

    My more non-obvious point, which eludes Searle and practically everyone else, is that the mind is not an algorithm in the standard sense, because it is physically embodied. Unlike an algorithm, it does not take inputs and compute an output, but participates in the world in a continuous and interactive fashion (causally and otherwise).

    But there is nothing stopping us from building computer-like machines that also participate in the world, such as robots. Such machines are not yet much like minds, but they aren't much like Turing machines either.

    Even if you ignore all that, as I expect you will, the Church-Turing thesis does nothing to help Searle's argument. You seem to know something about computers, you ought to be able to see the bugs in his creaky device.

    ReplyDelete
  45. Edward Feser. I just realized a bigger problem with the argument you give here:

    "A Frege pointed out, a proposition (like the proposition that snow is white) cannot possibly be identified with any sentence or set of sentences ("Snow is white," "Schnee ist weiss," etc.) because the proposition would remain true whether or not any sentence or indeed any language at all ever existed."

    How on earth could the statement "no language exists" possibly be true?

    ReplyDelete
  46. This is slightly off-topic from this thread, but I was thinking about intentionality, propositions, etc., and it occurred to me that I had no idea what A-T philosophers would say about the ontological status of propositions, probably since such is an issue primarily in contemporary metaphysics. I assume that Aquinas et al would say that propositions exist, but are not mind-independent? But then, how are they intersubjectively available?

    -Landon

    ReplyDelete
  47. Steve Ruble,
    You will find that Feser writes in Aquinas (page 137) that the causal processes that take place in a snake as he digests a meal are irreducible to the kinds of casual processes that take place as when snake slithers over earth or as when purely non-biologic events such as rainstorm occur.

    However, the casual processes are defined somewhat differently in Thomism so I can not be too sure but he seems to be saying that biologic casual processes are essentially different from non-biologic casual processes.

    Also see the discussion of immanent causation vs transeunt causation.

    ReplyDelete
  48. 'How on earth could the statement "no language exists" possibly be true?'

    I think I can venture a reply on Feser's behalf. The "statement" that "no language exists" couldn't be true, because that would be self-defeating; we seem to agree on this.

    On the other hand, the proposition that no language exists could be true even if there were no language, precicely because propositions are distinct from sentences. This, naturally, plays right into Feser's hands.

    Unless I'm reading you wrong, it seems that you're simply confusing propositions with sentences.

    ReplyDelete
  49. @Steve Ruble:

    "Nope, you've got it wrong. I know what I'm criticizing; I just don't find it remotely plausible, nor particularly interesting."

    Chuckle. Yeah, right. As I said: an argument from incredulity betraying a profound ignorance.

    "I mean, I understand that it's fun to play with semantic systems and to just see what you can work out from a set of axioms or concepts - I'm a programmer, I love doing that stuff - but there's a huge difference between doing that and actually trying to figure out what's going on in the real world. Ya'll seem to think that your word games should be able to constrain what can be truthfully said about the world, and that your word games are safe from dis-confirmation by anything in the real world. I don't think you can have it both ways."

    Metaphysics is not "play with semantic systems", but I do not expect you to recognize the difference neither am I inclined to educate you. Your scientistic non-sense has been amply debated in this very blog, and honestly, ATM I do not have the least patience for it.

    ReplyDelete
  50. @goddinpotty:

    "the Church-Turing thesis is about the equivalence of certain classes of computational problem. It says exactly nothing about the mind. Well, it does say that if the mind is an algorithm, then that algorithm is realizable by a Turing machine. I fail to see the relevance to the discussion. If the mind is an algorithm, then it has the properties of an algorithm, and if not, not."

    Are you being deliberately obtuse or just playing one to make a point? Your assertion was and I quote for the third time:

    "Searle is very confused. Unpacking his confusion is not all that simple, because his critque is somewhat valid when applied to a large class of AI programs, particularly those that date from around the time he floated this line of attack. But that does not mean they apply to all computational systems of all time."

    So what you are saying is that there is at least one computational system to which Searle's arguments do not apply. Since Searle's Chinese room experiment has a man inside a cubicle who obviously enough can simulate a universal Turing machine (minus the usual caveats, unbounded memory and time) ergo it follows that there is a computational system that is not modeled by a Turing machine, ergo the Church-Turing thesis is wrong. In other words, I am not claiming that the Church-Turing thesis says anything about the mind, I am saying your unargued claim has the implication it has since all computational machines are physical implementations of Turing machines. To spell it out so that even you can understand it: it is a roundabout way of saying your unargued claim is all bluster and nothing of content.

    "My more non-obvious point, which eludes Searle and practically everyone else, is that the mind is not an algorithm in the standard sense, because it is physically embodied. Unlike an algorithm, it does not take inputs and compute an output, but participates in the world in a continuous and interactive fashion (causally and otherwise).

    But there is nothing stopping us from building computer-like machines that also participate in the world, such as robots. Such machines are not yet much like minds, but they aren't much like Turing machines either."

    So now the source of Searle's confusion is that his arguments do not apply to all computational systems but the fact that computational systems are physical machines and can causally effect the world. The point is not "more non-obvious"; it is trivial and irrelevant to Searle's arguments.

    "You seem to know something about computers, you ought to be able to see the bugs in his creaky device."

    You are right, I do know something about computers. To expect that knowledge of computers will help one in seeing "the bugs in his creaky device" just betrays your own confusion and ignorance -- in fact, that is one of the reasons why I specifically formulated things in terms of Turing machines as they are a universal model for computation and thus, the precise details of the computing machine or knowledge of them is *irrelevant*.

    ReplyDelete
  51. @Landon:

    "This is slightly off-topic from this thread, but I was thinking about intentionality, propositions, etc., and it occurred to me that I had no idea what A-T philosophers would say about the ontological status of propositions, probably since such is an issue primarily in contemporary metaphysics. I assume that Aquinas et al would say that propositions exist, but are not mind-independent? But then, how are they intersubjectively available?"

    Not the most appropriate person to answer this as my knowledge is fragmentary and very incomplete, but here goes. The short answer is that the Form, a real existing extra-mental reality, that inheres in a given particular, is grasped by the intellect and lives in the mind as a universal, abstract concept. The *same* Form that inheres in the object also lives in the mind but in a different mode of being, since it has been abstracted away from the particulars in which it inheres. Thus the problem you raise is answered.

    ReplyDelete
  52. Arthur

    I think I can venture a reply on Feser's behalf. The "statement" that "no language exists" couldn't be true, because that would be self-defeating; we seem to agree on this.

    On the other hand, the proposition that no language exists could be true even if there were no language, precicely because propositions are distinct from sentences. This, naturally, plays right into Feser's hands.

    Unless I'm reading you wrong, it seems that you're simply confusing propositions with sentences.



    Feser was trying to argue that propositions aren't sentences. His reasoning only works if propositions aren't sentences. In other words, he's begging the question.

    In fact, it's worse than that. It only works if there are propositions that cannot be expressed by English sentences. Needless to say, the proposition expressed by the English sentence "snow is white" is not one of them.

    -not donald

    ReplyDelete
  53. grodrigues - The first and the last sentence are incorrect. The second most probably is.

    "An outside source of change is also necessary... if a potential could actualize itself, there would be no way to explain why it does so at one time and not another... So, no potential can actualize itself, and in this sense anything that changes requires something outside itself to change it." - "The Last Superstition", pages 54-55.

    Can you see where I got the idea that, as I wrote, "Nothing changes unless acted upon by something else, allegedly"? How, exactly, is that sentence 'incorrect'? What have I misunderstood?

    As to the "last sentence", where I wrote, "And isn't 'acting without being acted upon' practically the definition of free will?", on page 55 of TLS Feser goes on to state:

    "This is true even of animals, which seem at first glance to change themselves; for what this always amounts to is really just one part of the animal being changed by another part. The dog "moves itself" across the room, but only insofar as the potential motion of the legs is actualized by the flexing of the leg muscles, and their potential to be flexed is actualized by the firing of the motor neurons, and the potential of the motor neurons to fire is actualized by other neurons, and so on and so forth".

    But what of humans? If I have free will, I can choose things - I can change my behavior. I can actualize my potential power to speak, or to hold my breath. If I were doing that because something else acted on me, then how would my will be free?

    Again, I don't understand where my error is. Does Feser go on to cover this later in the book? I've looked up "free will" and "quantum mechanics" in the index, and skimmed the referenced pages, but my questions don't seem to be answered there.

    Things like Bell's Inequality and similar results provide strong evidence that QM events are indeed 'spontaneous' in the sense that, say, a nucleus decays without anything acting on it. To use Feser's words, it really does seem that there's "no way to explain why it does so at one time and not another". So while my "second sentence" may be incorrect, it doesn't seem so now.

    ReplyDelete
  54. Feser was trying to argue that propositions aren't sentences. His reasoning only works if propositions aren't sentences. In other words, he's begging the question.

    Feser's original argument (at 3:14) was that propositions can remain true even if language never existed, a premise that he assumed you believed too. Given what you said (at 8:05) you seem to recognize the problem too, and therefore implicitly agree with him. You asked, 'How on earth could the statement "no language exists" possibly be true?' The answer is that it couldn't be, and while the statement that "no language exists" couldn't be true, the proposition that "no language exists" could be. In other words, propositions are a solution to the very problem you posed. Whether deliberately or not, you seem to have put forward a mostly-complete argument for propositions yourself.

    Even if Feser was begging the question, it does seem that he was begging the question in favour of something you actually believe, whether you noticed it or not.

    'In fact, it's worse than that. It only works if there are propositions that cannot be expressed by English sentences.'

    I don't think that's what Feser said. It isn't required that there are some inexpressible propositions, completely independant of language, only that propositions aren't reducible to the sentences that express them. As Feser said,

    'Nobody is saying that it must be expressible apart from language. The claim is that it isn't reducible to bits of language considered merely in terms of their physical properties.'

    In other words, propositions need to be distinct from their expressions, but not entirely detached. Sentences express propositions.

    ReplyDelete
  55. By the way, you can in fact coherently say that "snow is white" could be true even if the English language doesn't exist, but as we saw before, allowing a distinction between sentences and propositions doesn't do the trick. What actually works is removing the requirement that hypothetically true statements hypothetically exist.

    As a side note, this discussion gets very close to the core of how Aristotelian arguments for God work. Like most arguments for God, they are arguments from incredulity:

    "I can't imagine how things would work if there wasn't a God. Therefore God."

    except the Thomist definition of God is a mind that is thinking about the universe. Thus the argument becomes.

    "I can't imagine a world that is not being imagined by a mind. Therefore the world is being imagined by a mind."

    The premise absolutely, positively MUST be true, but it still doesn't justify the conclusion. As a bonus, the God the Thomist sees in his model of the world is none other than himself, complete with every regressive hateful prejudice he might have.

    -- not donald

    ReplyDelete
  56. you can in fact coherently say that "snow is white" could be true even if the English language doesn't exist

    ack. Even if the Enlish language DIDN'T EXIST.

    Of all the places to forget the subjunctive...

    -not donald

    ReplyDelete
  57. "I can't imagine a world that is not being imagined by a mind. Therefore the world is being imagined by a mind."

    The beauty of these purely metaphysical arguments (meh) are that there is no way to verify or falsify. Just faith in the belief.

    The non material 'soul' is exempt from study.

    ReplyDelete
  58. @ not Donald:

    As far as I know Aquinas nowhere says "I can't imagine A - therefore B". On the contrary, Aquinas tries to prove "no A - therefore B.

    Not everybody accepts his proof, but that's a different matter.

    T.H.

    ReplyDelete
  59. "I can't imagine a world that is not being imagined by a mind. Therefore the world is being imagined by a mind."

    Can you quote anyone on this?

    ReplyDelete
  60. Arthur said...

    "I can't imagine a world that is not being imagined by a mind. Therefore the world is being imagined by a mind."

    Can you quote anyone on this?


    No. It is my own diagnosis of what's going on. Of course neither Thomas nor Feser would put it like that, because it lays bare both the problems with such arguments and why such arguments seem so convincing.

    They assume imaginability by little bits.

    We can't imagine infinitely many things, hence arguments about infinite regresses.

    Different parts of the brain process shape and color/texture,
    hence the distinction between form and substance.

    The brain circuits involving goal directed reasoning are often reused to simulate the natural world. Hence, even inanimate objects are conceived of as being replete with "function."

    I don't think any of this was done intentionally, mind you, but we can in fact see that the God of the Thomist's world view is in fact the Thomist himself (this is pretty much true of all theists) as illustrated by this study: http://www.pnas.org/content/early/2009/12/01/0908374106.abstract

    --not donald

    ReplyDelete
  61. "Can you quote anyone on this?"

    "No."

    In other words, you're just blowing smoke. It may be "your own diagnosis of what's going on", but where did that diagnosis come from? We deserve analysis, not insinuation.

    I also don't see the point of the study you linked to. Whatever it shows, it simply psychologizes theism rather than engaging with its arguments. You'd be better off quoting directly from, say, Aquinas, and explaining what's wrong with what he says. If you can't do that, you're simply wasting everyone's time.

    ReplyDelete
  62. Arthur

    "Can you quote anyone on this?"

    "No."

    In other words, you're just blowing smoke. It may be "your own diagnosis of what's going on", but where did that diagnosis come from? We deserve analysis, not insinuation.


    so your idea of analysis is argument from authority, rather than what I provided, which was supporting evidence.

    I also don't see the point of the study you linked to. Whatever it shows, it simply psychologizes theism rather than engaging with its arguments. You'd be better off quoting directly from, say, Aquinas, and explaining what's wrong with what he says. If you can't do that, you're simply wasting everyone's time.

    Perhaps you haven't noticed, but I've been picking out unfounded premises from arguments like those of Feser and Aquinas all day. And I CAN quote as high an authority as Aristotle Scholar Sir Anthony Kenny to support the position that Aquinas's arguments don't work (since as we established above, Argument from authority is your idea of analysis.) Frankly however, addressing specific instances gets boring after awhile. It is the general principle of the thing that brings true understanding.

    ReplyDelete
  63. Some points with regard to Turing machines:

    That computers interact with the real world (as they always have, even when Searle came up with the Chinese room) is irrelevant to whether they can be modeled by a Turing machine. That interaction is precisely what a Turing machine models with inputs and outputs. Any computational interaction a computer has with the real world involves data being put into a form that the computer performs computations on, and that computation can be modeled with a Turing machine.

    That computers could have continuous inputs and outputs is irrelevant as well. A computer of any speed, receiving inputs and generating outputs just as fast and as continuously as you like, can be modeled by a Turing machine.

    That real systems may involve multiple processors, or multiple networked computers working in tandem, is also irrelevant. Any computation or set of computations that can be modeled by multiple Turing machines can be modeled by a single one.

    If you want to say that computers could instantiate mind, but you don't want to say that the mind can be fully modeled by a Turing machine, the only options here are to show that the Church-Turing thesis is false, and that computation cannot be fully modeled by a Turing machine after all (which would make you the greatest mathematician and computer scientist of all time), or to say that computers can do something *other than* computation which captures mind (which is an implicit denial of computationalism).

    ReplyDelete
  64. @Ray Ingles:

    "The first and the last sentence are incorrect. The second most probably is.

    "An outside source of change is also necessary... if a potential could actualize itself, there would be no way to explain why it does so at one time and not another... So, no potential can actualize itself, and in this sense anything that changes requires something outside itself to change it." - "The Last Superstition", pages 54-55.

    Can you see where I got the idea that, as I wrote, "Nothing changes unless acted upon by something else, allegedly"? How, exactly, is that sentence 'incorrect'? What have I misunderstood?"

    Sorry, my bad here. It is not that it is incorrect strictly speaking, is that it lends itself to all sorts of confusions and equivocations. Saying that "no potency can actualize itself" may or may not be the same as "Nothing changes unless acted upon by something else" depending on what you mean by something else.

    "But what of humans? If I have free will, I can choose things - I can change my behavior. I can actualize my potential power to speak, or to hold my breath. If I were doing that because something else acted on me, then how would my will be free?

    Again, I don't understand where my error is. Does Feser go on to cover this later in the book? I've looked up "free will" and "quantum mechanics" in the index, and skimmed the referenced pages, but my questions don't seem to be answered there."

    As far as I can remember, Prof. Feser does not deal with Free Will in TLS. The Thomistic account of Free Will is somewhat complicated -- suffice it to say that your own gloss is incorrect.

    "Things like Bell's Inequality and similar results provide strong evidence that QM events are indeed 'spontaneous' in the sense that, say, a nucleus decays without anything acting on it. To use Feser's words, it really does seem that there's "no way to explain why it does so at one time and not another". So while my "second sentence" may be incorrect, it doesn't seem so now."

    Oh brother, not another rant on how QM supports the idea of uncaused events... Without going into the utter absurdity and irrationality that is the positing of uncaused events, and sticking to physics as much as possible:

    1. If you want to support the idea that events at the (sub)atomic level are uncaused than you have to explain to me exactly what you mean by cause writ large. Since this is not a question of physics, QM will not help you in the least.

    2. Related to 1. but slightly different, QM is an abstract mathematical formalism whose job is to correctly predict the correlations measured in experiments. Making ontological statements from theoretical constructs is to confuse the map for the territory. Argumentation is needed, and since the matter is at bottom, a metaphysical one, QM alone cannot do the job for you -- although it can certainly inform the arguments.

    3. Since there are fully causal and deterministic interpretations of QM you have to justify your particular pick (most probably, Copenhagen). Since this cannot be decided empirically, at least not yet, physics will not help you.

    4. Bell's inequality does not support "spontaneous" events. It is a no-go theorem whose significance lies elsewhere. And there are loopoholes in it that can be exploited to regain full deterministic, local causality -- although some of the cures are worse than the disease.

    I do wonder if all these people that have suddenly appeared with mostly unsubstantiated claims think that we here are all ignorant of physics or computer science in the same way they are ignorant of what they propose to criticize? Projection is a bitch, but this is just plain ridiculous.

    ReplyDelete
  65. "...so your idea of analysis is argument from authority..."

    Not at all. Where do I provide an argument from authority in my posts? Let me guess; you can't quote me on that, nor explain why you interpreted me that way. You may be surprised to hear that I don't even (yet?) believe Aquinas. You don't need to be a Thomist to oppose misunderstandings of Thomism, or identify bad arguments.

    You also don't seem to understand the problem with the study you linked to. It wasn't "supporting evidence" at all; it was irrelevant. As I mentioned, regardless of what the study shows, it only shows us something about theists' beliefs, not arguments for theism.

    I'll say it again. Quote Aquinas' words directly. Explain your interpretation of them. State clearly what fallacy was committed. If you can't do that, you have nothing to say worth saying.

    If you think that you've landed some killer objetion to Thomism, could you restate it? It gets tricky telling all these anonymouses apart.

    ReplyDelete
  66. @ Anonymous 5:15 - not donald,
    Your argument against theism from imagination works even better against atheism.

    I cannot imagine what god is like.Therefore god does not exist.

    The only things I can imagine are material things. Therefore, immaterial things like the soul and god do not exist.

    And one more for good measure,(the one you actually use in your post):

    If I cannot verify something with empirical evidence and science, it does not exist.

    All of these arguments are obvious fallacies. So the conclusion which follows and parallels your view of of theism is that atheism is a bad joke.

    If you think that this style of argumentation does not work against atheism, then you you should stop pretending that it works against theism. Arguments for the existence of God and the soul are grounded in reason and our understanding of being/existence, not in the imagination.

    ReplyDelete
  67. grodrigues - Hmm. So, I may or may not grasp Feser's point about act/potency. And there *is* an account of free will, but not in TLS. Well, okay.

    I know that there have been attempts to rescue local determinism from the implications of Bell's Theorem, but I've found them about as convincing as... well, as y'all find computational accounts of cognition, I suppose. (Similarly, GR seems most naturally interpreted in light of a "B series" model of time.) I suppose I'll have to set this aside for now as I keep reading TLS.

    BTW, from the earlier comment - "The existence of non-euclidean geometries is irrelevant to the truth or not of Platonism or any other form of mathematical realism."

    Well, sort of. But considering that the geometries arise from conflicting axioms (in this case, the parallel postulate) the way in which they exist could be looked at as violating the Law of Contradiction. I mean, both hyperbolic and elliptical geometry can't be true in the same region of space and time. It points to a kind of 'multidimensional Platonic realm', it seems to me. Speaking of which...

    It seems to me that imaginary numbers argue against Feser's point - insofar as it applies to Aristoteleanism vs. Platonism as I've read so far. Feser's account of A-T allows for forms existing without matter, I assume? Does TLS get into that?

    ReplyDelete
  68. Arthur.

    I already have more than enough reason to reject Thomist arguments for God. They claim to be deductive arguments and are based on unfounded premises. Furthermore, it's not even clear what they are supposed to prove if not a scientific claim.

    Well founded deductive conclusions have a name. Mathematical theorems. Proofs of such theorems, even fairly complicated proofs, tend to become accepted very quickly and overwhelmingly. (even ones, like the proof that some square-roots are irrational, that destroyed sacred cows of one sort or another.) Aquinas and his defenders have had 800 years to shore up their arguments, and they've convinced essentially no one. In other words, if you want to claim you have a valid deductive argument for something, convincing a significant number of mathematicians does not seem like an unreasonable burden of proof.

    So, once it becomes clear, for whatever reason, that these proofs are not as undeniable as their defenders claim they are, all that's left to do is to analyze the source of the confusion. The discipline that does this is psychology. Hence the paper.

    Oh well. I suppose I'll try being non-anonymous. It's rather a pain trying to sign all my posts

    --not donald

    ReplyDelete
  69. "I already have more than enough reason to reject Thomist arguments for God."

    Do you now? Then why do you have so much difficulty quoting Aquinas' actual words and explaining what's wrong with them? It's easy to boast that you have "more than enough reason", harder to show it.

    "They claim to be deductive arguments..."
    Well, yes. Many of them are deductive arguments. What's wrong with that?

    "...and are based on unfounded premises."
    Like what? Again, you keep boasting about these "unfounded premises" but can't be more specific. Just one would do for now.

    "Furthermore, it's not even clear what they are supposed to prove if not a scientific claim."
    Golly, what other kinds of knowledge could there be other than scientific knowledge? After all, all knowledge is scientific... right? Oh, wait, that's a self-defeating proposition.
    Also, just because it's "not clear" what kind of statement they are hardly makes them bogus by itself. I'm sure Feser and his followers can explain metaphysics to you better than I can. As it happens, I have doubts about metaphysics myself, but I wouldn't dream of arguing from ignorance and saying that just because it's "not clear" what kind of statements they are that they are meaningless or false. If it's "not clear" to you what Thomism is about, I recommend that you try to understand it rather than reject it.

    "Aquinas and his defenders have had 800 years to shore up their arguments, and they've convinced essentially no one."
    Aquinas' arguments are unpopular, you say? I knew that already. It's a point that gets repeated fairly often on this blog. The trouble is that it really does not matter how many people are convinced by the arguments, what matters is the content of the arguments themselves. Appeal to (un)Popularity will get you nowhere. As Feser repeatedly shows on this blog, most people are woefully ignorant of what Aquinas actually said.

    "So, once it becomes clear, for whatever reason, that these proofs are not as undeniable as their defenders claim they are, all that's left to do is to analyze the source of the confusion. The discipline that does this is psychology. Hence the paper."
    You seem to be backpedalling a little here (not that there's anything wrong with that). The original point of the paper was clearly to support your beliefs about how most "arguments for God work", not simply to psychoanalyse theists. If all it's intended to do is explain how theists think, I don't see the point in putting it forward. When you have a convincing argument that Thomism is bogus, then we can talk about psychoanalysing Thomists. Until then, such things are premature.

    I know I keep saying this, but I'll say it again. You're blowing smoke. You boast that you have "enough reason" to reject Thomism, but don't tell us what that reason is. All you can offer are appeals to its unpopularity. You boast about "unfounded premises", but can't name any. There isn't much evidence in your posts that you've so much as read anything Aquinas has to say, let alone understood it.

    I've already told you what you can do to convince me otherwise. Quote Aquinas. Interpret him. Explain the fallacy. If you can't do that, your "enough reason" is just so much bluster.

    ReplyDelete
  70. @Ray Ingles:

    "I know that there have been attempts to rescue local determinism from the implications of Bell's Theorem, but I've found them about as convincing"

    So do I (note: actually I would need to qualify this, but never mind), but local determinism was not the original issue of contentioon, but uncaused events.

    "GR seems most naturally interpreted in light of a "B series" model of time."

    Once again, you make the same mistake and let the abstract formalisms guide the ontology. What it "seems" to you or me, is irrelevant. If you want to make an argument that GR provides evidence for "four-dimensionalism" then simply waving your hands is not enough -- make the argument. By the way, "four-dimensionalism" has many, many problems. For a critique see for example, D. Oderberg "The metaphysics of identity over time".

    "The existence of non-euclidean geometries is irrelevant to the truth or not of Platonism or any other form of mathematical realism.

    Well, sort of. But considering that the geometries arise from conflicting axioms (in this case, the parallel postulate) the way in which they exist could be looked at as violating the Law of Contradiction. I mean, both hyperbolic and elliptical geometry can't be true in the same region of space and time. It points to a kind of 'multidimensional Platonic realm', it seems to me."

    You have just provided the answer. Look, what is the common axiomatic framework for the vast majority of mathematics? ZFC. A Platonist could say that ZFC is an (incomplete by Goedel) formalization of the "real" set-theoretic universe. But since the ground-breaking work of Cohen on forcing and the independence of CH in the 1960's, and the ensuing plethora of models of set theory and new set-theoretical axioms, some mathematicians like J. D. Hamkins posit a set-theoretical multiverse, that is, there is no unique set-theoretical universe but a whole plethora of them organized into a structure with some characteristic properties. But such a conjecture even if true, presents no problem for the would-be Platonist because it is then the Multiverse that is the "real" thing -- as Hamkins himself defends. See http://arxiv.org/abs/1108.4223. Same comments with much more propriety can be made about geometries, which can all be formalized in ZFC (and in most cases in tiny fragments of it). Either way, your comments say absolutely nothing about the validity or invalidity of Platonism or any form of mathematical realism. And space-time has nothing to do with it either.

    "Feser's account of A-T allows for forms existing without matter, I assume? Does TLS get into that?"

    Once again "forms existing without matter" is prone to equivocation; the correct way to put things is that there are no uninstantiated essences. Nevertheless, although I would have to reread the book, the issue is at least adumbrated when knowledge and the immortality of the soul are discussed.

    ReplyDelete
  71. >I'll say it again. Quote Aquinas' words directly. Explain your interpretation of them. State clearly what fallacy was committed. If you can't do that, you have nothing to say worth saying.

    Have I mention how much I admire rational skeptics/atheists/agnostics/non-believers etc?

    Guys who use common sense and reason vs kneejerk dogmatism and Dawkins like profundity?

    You rock Arthur!

    Thank you.

    ReplyDelete
  72. Anytime, guys. Nice to know someone's reading.

    ReplyDelete
  73. Here're some unfounded premises. At least one for each of the five ways:

    1)"whatever is in motion must be put in motion by another."

    which does not seem to follow from the preceding text unless the italicized words are added to the following quote, making the quoted text both doubtful and unfounded:

    "nothing can be reduced from potentiality to actuality in some respect , except by something in a state of actuality in the same respect"

    2)"if there be no first cause among efficient causes, there will be no ultimate, nor any intermediate cause."

    or more precisely the claim that it follows from

    "to take away the cause is to take away the effect."

    3)"that which is possible not to be at some time is not."

    4)"“more” and “less” are predicated of different things, according as they resemble in their different ways something which is the maximum."

    (only supported by example, which isn't sufficient in deductive argument. The example is doubtful too.)

    5)"things which lack intelligence, such as natural bodies, act for an end ... so as to obtain the best result"

    or that any of the above can be deduced from the fact that the bodies act "always, or nearly always, in the same way"

    or
    "whatever lacks intelligence cannot move towards an end, unless it be directed by some being endowed with knowledge and intelligence"

    I tried to only go for premises which were not just unfounded, but rather doubtful as well. There appear to be many more unstated premises required to make these arguments work. I don't think I saw a single therefore which was followed by a statement that strictly followed from the text before it, but I tried to read charitably.

    There's some other supporting text in the Summa (most notably responses to objections.) But responding to objections is really more a tool from inductive reasoning than deductive reasoning. A deductive argument should stand on it's own.

    Congratulations Arthur, you have just made me waste my time verifying a well known, and rather obvious, fact. Did you really doubt that there were some unfounded premises in there?

    ReplyDelete
  74. "Congratulations Arthur, you have just made me waste my time verifying a well known, and rather obvious, fact. Did you really doubt that there were some unfounded premises in there?"

    Gathering evidence is not a waste of time, Ray. And yes, I really did "doubt that there were some unfounded premises in there". What can I say? I'm a sceptical thinker, and I don't believe something until I see evidence for it. I'm sure you can sympathise.

    I also love your comment that Thomism's weaknesses are "obvious". If the Thomists here don't believe it, and even I don't believe it, how "obvious" could it meaningfully be? Appeal to the "obvious" is the oldest trick in the book.

    Granted, the important premises you've quoted may seem doubtful to you, but I'm sure a Thomist could defend them. Just because you, personally, find the premises doubtful is only a beginning. Have you presented your doubts to a Thomist in the form of an argument? You don't even sound interested in trying. Or would a debate simply be another waste of time given how "obvious" all this is?

    I also don't understand your comment that, "responding to objections is really more a tool from inductive reasoning than deductive reasoning. A deductive argument should stand on it's own." Surely a deductive argument can meaningfully be defended against, say, misunderstandings of the terms (as Aquinas' arguments often are), or any other kind of doubt. It seems very odd to demand that every deductive proof must be immediately convincing if it is to be convincing at all. Stranger yet, you allude briefly to "other supporting text in the Summa" as a way of "responding to objections", but don't seem remotely interested in whether your doubts are addressed there.

    I won't pretend that I could give a satisfying defence of all the premises you find doubtful. Like I said, I'm not even a Thomist and there are others here who could do a much better job than me. However, I don't think that this is very significant. If you find the premises in Thomistic arguments "doubtful", the rational thing to do is ask Thomists about them and try to understand them, not to leap to the conclusion that they must be bogus and start psychoanalysing anyone who disagrees with you. I have doubts about Thomistic metaphysics too, but I wouldn't dream of jumping to the conclusion that they are therefore "unfounded".

    You're slowly getting there. You've finally quoted Aquinas' actual words, but beyond the Argument from Personal Incredulity that you find them "doubtful", you don't seem to have much to say against them.

    ReplyDelete
  75. @Arthur:

    "You're slowly getting there. You've finally quoted Aquinas' actual words, but beyond the Argument from Personal Incredulity that you find them "doubtful", you don't seem to have much to say against them."

    What Ray engaged in is the oldest trick in the book and a textbook example of intellectual dishonesty. The Summa was written as a manual for students where the arguments are given only in a very succint form; some are expounded at greater length elsewhere; all presuppose a considerable background in Scholastic metaphysics. Let us see if Ray can explain the primary division in being, that between act and potency, the four causes, the difference between causal series per accidens and per se, etc. and etc.

    It is like picking an informal description of the Banach-Tarski paradox (a pea can be divided in a finite number of pieces, that when moved around by translations and rotations and then reassembled make up two peas) or of space-filling curves (a zero-dimensional point can pass through every point of a 3d cube in a finite amount of time) and then object that the mathematical proof must be wrong based solely on the informal description and because one finds the conclusion "implausible".

    If Ray *really* wants to take a shot at it, then go and read a decent book, say "Aquinas" by the gracious host of the blog -- no, I do not know the man, neither will I receive any royalties. Or if he is not willing to forfeit the cash let him slog through http://www.thesumma.info/reality/index.php (warning: tough reading) instead of look at what was always meant as a summary and then declare victory without showing the least tincture of understanding.

    ReplyDelete
  76. Um. Arthur. You do realize that unfounded is a weaker claim than false (and in fact weaker than doubtful.) Right?

    Perhaps you've been hanging out around Feserites too long -- they seem to make claims that come dangerously close to equating the two.

    Hopefully you know that there are true statements which are not provable. That IS a theorem, by the way.


    As for accusations of backpedaling. I think it's clear that my original post was attempting to explain why Aquinas's arguments seem convincing to believers -- which is a psychological problem. I take it as already established that they don't work. The argument from popularity is fine for any reasonably well known argument claiming to be a deductive argument. -- There is copious evidence that valid deductive arguments are reasonably easy to verify, and become widely accepted among mathematicians quickly. Do you deny this, and if so, what do you think the point of peer review in math journals is?

    Oh, and if anyone upthread is still claiming that their mind cannot be simulated by a Turing machine. Solve the halting problem already or shut up. (And for pedants out there, I am aware there are Turing degrees between 0 and 0'. I'll accept a problem of one of those Turing degrees too.)

    ReplyDelete
  77. Ray not donald

    Odd that you should take that superior tone with Arthur when Arthur is, in fact, quite right: these are not unfounded premises but rest directly on primary principles of reason so that it's impossible to reject them without at the same time calling into question the validity of human reason (this may or may not be a problem for you).

    I hope you take grodrigues's advice and you'll find his link well worth your effort.

    Still not sure on the relevance of your study explaining believers' "psychological problem"s. Thomists would say that the ultimate source of all error is the Will, and egocentricity is not normally numbered among the chief proximate causes (your linked study notwithstanding). With machinephilosophy in mind, I'd suggest you don't grant nonbelievers (like yourself?) an unprincipled exemption and examine:

    1. Your prejudices
    2. Any imperfect teaching or false information you've received regarding facts and principles
    3. Your confusion of ideas, whether from dullness or inattention
    4. Your impatience to arrive at a conclusion by being too vain to acknowledge ignorance.

    That's a study I'd be interested in reading. Good luck.

    ReplyDelete
  78. grodrigues

    Let us see if Ray can explain the primary division in being, that between act and potency, the four causes, the difference between causal series per accidens and per se, etc. and etc.

    Yawn. None of these terms are unfamiliar to me. I'm not going to provide definitions, though; if I really wanted to fool you, I could just look them up, and asking me to prove my familiarity when I have given you no reason to doubt it is a sign of bad faith and an attempt to waste my time.

    If you want to claim the statements I provided are not unfounded, please provide links to the part of the Summa that supports each of them through valid logical deduction from uncontroversial premises.

    ReplyDelete
  79. Gyan,

    "You will find that Feser writes in Aquinas (page 137) that the causal processes that take place in a snake as he digests a meal are irreducible to the kinds of casual processes that take place as when snake slithers over earth or as when purely non-biologic events such as rainstorm occur."

    I got the impression that he was making similar claims at several points in The Last Superstition. Like you, I'm nevertheless unsure about how Feser thinks that the difference between these "causal processes" actually plays out in real life. What I'm trying to pin down is this: are the Thomists actually claiming that the physical particles which make up a human behave according to different rules than those physical particles which make up, say, a rock, by virtue of the human form, or do the particles in humans follow all the same rules as particles elsewhere in the world? Unfortunately, no one seems to be willing to answer this rather simple question.

    Of course, as an Anonymous pointed out earlier, metaphysicians are wary of making claims which could potentially be empirically dis-confirmed.

    ReplyDelete
  80. This comment has been removed by the author.

    ReplyDelete
  81. Ray not donald

    First, you told Arthur that you "tried to only go for premises which were not just unfounded, but rather doubtful as well[,]" now you are yawning at grodrigues for pointing out that believing those premises are unfounded is a reason to doubt your familiarity with the subject matter.

    It's not possible to prove the principle of causality, just as it's not possible to prove the principle of contradiction. Please just read grodrigues's link to see how you get from these to your premises.

    Bear in mind that if your next move is to try and reject both principles you're not just throwing out five classical proofs of God's existence but all human reason and knowledge too. Of course I'm sure you'll try to convince us that QM compels you...

    ReplyDelete
  82. @Ray:

    "Hopefully you know that there are true statements which are not provable. That IS a theorem, by the way."

    No, it is not. If you are going to quote mathematical theorems at least make an effort to quote accurately.

    "Oh, and if anyone upthread is still claiming that their mind cannot be simulated by a Turing machine. Solve the halting problem already or shut up."

    What a devastating refutation! I suggest you write it down -- it is only a couple of sentences after all -- and send the produce of your genius for peer-review in a top philosophy journal as you have solved a long-standing problem and decidedly refuted those deluded fools (in which sadly, I must count myself) that hold that computational accounts of the mind are false.

    "If you want to claim the statements I provided are not unfounded, please provide links to the part of the Summa that supports each of them through valid logical deduction from uncontroversial premises."

    Read my post February 28, 2012 2:18 PM again.

    I also note that in response to Arthur you said that "unfounded" is much weaker than doubtful, according to you a distinction "Feserites" seem to overlook, now you ask for "links to the part of the Summa that supports each of them through valid logical deduction from uncontroversial premises". Methinks, maybe you are a "Feserite" in disguise.

    As far as the uncontroversial qualifier, read Jack Bodie's post above. Nevertheless, let me add we are talking about Philosophy here, not Mathematics. And even in Mathematics, axioms, in the informal sense of "uncontroversial premises", can actually be the subject of hot controversy. Are you suggesting that your own metaphysical premises, whatever they are, implicit or explicit, are uncontroversial?

    ReplyDelete
  83. Ray (not donald),

    You need to refine your acquaintance with the English language; 'unfounded' can mean something either weaker or stronger than 'false', depending on the context, since it can in ordinary usage mean 'not yet established' (which is weaker) or 'false and groundless' (which is stronger). There was no way for Arthur to tell which you meant from your very vague comment.

    I'm amused that you think that "If X has no solution for the halting problem, X is a Turing machine" is true, which is what your challenge implies. Setting aside the fact that the halting problem for Turing machines is undecidable whether one is oneself a Turing machine or not (Turing's proof does not depend on who is doing the proving); and setting aside the fact that the halting problem for a class of things that were not Turing machines, if that is what you meant, would have to be precisely defined and not vaguely waved at with the label 'the halting problem' since the halting problem was originally defined entirely in terms of Turing machines; it is still the case that there's no reason to think that everything that's not a Turing machine would actually have a solution to the halting problem, given that there's any number of reasons one might not have an algorithm. The assumption of the challenge is, shall we say, 'unfounded'.

    I find your question especially amusing because it's a trivial question:

    What I'm trying to pin down is this: are the Thomists actually claiming that the physical particles which make up a human behave according to different rules than those physical particles which make up, say, a rock, by virtue of the human form, or do the particles in humans follow all the same rules as particles elsewhere in the world?

    Everybody reasonable holds that the physical particles which make up a human behave according to different rules than those that make up a rock; for instance, one of the things that the particles in my body do that the particles in a rock don't is drive to the library while obeying city traffic laws. Surely you would agree that (1) the rules of poker partly explain some of the behavior of human beings at a poker table and (2) the water molecules in the ocean don't exhibit behavior that is even partly explained by the rules of poker. Perhaps the reason you have difficulty getting answers to your questions is that your formulate your questions badly.

    ReplyDelete
  84. @grodriguez -- You are imputing your own (wrong) interpretation to what I said, so naturally you are confused. When I said that Searle's critique was "somewhat valid", I did not mean that it applied or did not apply to some formally defined class of problems. What I did mean is that Searle's points are somewhat valid when applied to a certain kind of practice within AI, which was in the habit of computing with ungrounded symbols with suggestive names and thus systematically confusing meaning-for-the-computer with meaning-for-the-programmer. Insofar as AI did that, then Searle's critique had some bite. However, that critique does not apply to all conceivable artificial machines that might be built, and indeed AI has tried to reform some of these practices from within, with partial success.

    Your responses seem to be degenerating into content-free insults and misinterpretation, so this will be my last word on the subject unless you manage to say something interesting.

    ReplyDelete
  85. @goddinpotty:

    "Your responses seem to be degenerating into content-free insults and misinterpretation, so this will be my last word on the subject unless you manage to say something interesting."

    Fine by me. Since you have not addressed the responses put forward, more fully fleshed by Deuce, in fact you insist on the exact same mistakes, your future silence seems quite adequate and fitting as you yourself are not saying anything remotely interesting.

    ReplyDelete
  86. grodrigues

    You got me on the Godel's theorem oversimplification. "Statement" can be defined to include statements about the natural numbers, and "provability" may be defined to require that it be done from a finite number of axioms, but I'm still leaving off the possibility that the language in which my oversimplification of Godel's theorem is defined is inconsistent. Ok.

    Brandon

    You are either misinterpreting me or incorrect on the halting problem. No class of machines can solve the halting problem on itself. But it's perfectly consistent to posit a machine of Turing degree 0', i.e. one that can solve the halting problem for Turing machines. http://en.wikipedia.org/wiki/Oracle_machine
    Further, you are eliding the distinction between "X is a Turing machine" and "X can be simulated by a Turing machine"

    ReplyDelete
  87. Steve,
    I don't think Thomists believe in the atomic hypothesis.
    To them, a living body is a substance in its own right and its nature, esp its final causes is not determined by its material constituents.

    So a cat is a substance just like a lump of iron and while certain properties
    of a lump of iron are determined by its material constituents, certain properties of the cat are determined so, but not all.

    I would like if the local experts could critique my statements and point out the error that I might have made in defining the Thomist position.

    ReplyDelete
  88. Gyan,

    Yah, the claim that Thomists have doubts about atoms resonates with me - I seem to remember reading something by Oderberg in which he denies the existence of elementary particles - but I find it hard to believe that anyone would actually fail to believe that we (and almost everything else) are made up of atoms. In addition, I've had arguments with Thomists (or devil's advocates of Thomism, I can't remember how dguller positions himself) in which they've claimed that the difficulty physicists have explaining why very small things act in the ways they do should be taken as evidence for the legitimacy of teleology as a method for constructing ontologies, so I don't think Thomism is intrinsically opposed to atomism.

    I think the problem is that Thomists want to claim that any attempt at reductionism is metaphysically doomed from the get-go, but they're also smart enough to realize that the observable world is pretty much made of atoms, and that the behaviors of atoms are pretty well understood. They also have a desire to avoid giving the impression that they believe in magical phenomena (viz. the Thomist revulsion towards any suggestion of Cartesian dualism and its interaction problems). And, of course, they want to make the consideration of holistic forms and final causes the predominant method of analysis when it comes to interesting aspects of the world.

    That all adds up to a worldview that has some major issues when those who hold it are forced to interact with actual facts about the world. That is why, I suspect, they tend to respond to criticism with as much ad hominem bluster as they can muster. I can only hope that they'll respond more helpfully to your much more congenial request than they have to my rather abrasive criticisms.

    ReplyDelete
  89. “…The laws of physics are conceptual tools we have discovered in our collective efforts to impose coherence on how the world impinges on us, and ultimately the world gets through to each of us only through our conscious perceptions. This, of course, is as true of classical physics as it is of quantum physics. What makes quantum physics special
    is that it forces us or ought to force us to acknowledge this to be the character of physical law, while in classical physics we could and did fool ourselves into thinking that
    the abstractions we created to help us organize our perceptions had an independent existence of their own…”
    N. David Mermin,
    Am. J. Phys. 75 3, March 2007

    “...The reality of atoms and the content given to this notion have remained issues ever since the concept of an atom was introduced. This question obtains a new content by the identification of reality with the world of experience and the recognition that, accordingly, there is no world of atoms underlying the macroscopic world. Hence, in the presentation of quantum mechanics based on genuine fortuitousness, an expression like ``the world of matrix variables'' has been avoided, since it might convey the notion that something exists beyond the world of experience…”
    Ole Ulfbeck and Aage Bohr
    Foundations of Physics, Vol. 31, No. 5, 2001

    ReplyDelete
  90. "that the behaviors of atoms are pretty well understood."

    Steve,
    I would take issue here. The behavior of atoms is not understood. We do not understand why two positive charges should repel each other, for instance.

    I do think that the Thomism theory of mind is better than any other but their theory of animal digestion, I do not know.

    This point generally does not quite come up for discussion here which is mostly about theories of mind.

    One thing to keep in mind that in Thomism, the matter also has non-quantifiable aspects.

    ReplyDelete
  91. "in our collective efforts to impose coherence on how the world impinges on us"

    Does that mean that the world is not coherent and only our minds impose coherence?

    ReplyDelete
  92. @Steve
    >I seem to remember reading something by Oderberg in which he denies the existence of elementary particles - but I find it hard to believe that anyone would actually fail to believe that we (and almost everything else) are made up of atoms.

    Steve you should know by now terms used by the ancients might have a different meaning than those same terms employed by moderns. It's one of Dr. Feser chief themes(i.e. Aristotle on motius, motion, change, etc).

    Do you believe units of matter made of Electrons, Protons and Neutrons are "Atoms"?

    Democritus the world's first materialist/Atheist philosopher would not have thought so.

    For Him an "atom" was a elementary particle so small as too contain no void.

    Clearly atomic units of matter don't fall under that category.

    Indeed if there is anything in physics that might resemble an "atom" in the sense taught by Democritus I would nominate a Naked Singularity.

    (but of course matter is not made up of those but a NS wouldn't contain any void since it's infinitely dense).

    You will have to give us the precise Paper or writing but I would bet that is what Oderberg meant.

    We are not made of "Atoms" but we are composed of units of matter containing Electrons, Protons and Neutrons. As well as other things.

    ReplyDelete
  93. BenYachov,
    Yes the atoms of Heisenberg are not the atoms of Rutherford are not the atoms of Democritus but the point Steve made remains.

    "are the Thomists actually claiming that the physical particles which make up a human behave according to different rules than those physical particles which make up, say, a rock, by virtue of the human form, or do the particles in humans follow all the same rules as particles elsewhere in the world?
    ---------
    I would replace "human" by a "living body"

    ReplyDelete
  94. Anonymous quote-poster,

    I like the Mermin quote, but I can't see many Thomists agreeing with it. I'm confused by the second quote, because (thanks, internet!) Aage Bohr apparently won the Nobel for work on the structure of the atomic nucleus, which to my mind makes it less likely that he really intends to cast doubt on the existence of atoms in the article you're quoting.

    Gyan,

    I'm sorry, I didn't mean to claim that all the "why" questions about atoms had been answered ("why two positive charges should repel each other, for instance"), but rather that the actual observable behaviors of atoms and their constituent parts (that two positive charges repel each other, for instance) have been thoroughly documented, and that the current mainstream theories describing atoms, (and neutrons, protons, and electrons) have been as thoroughly validated by empirical observations as one could reasonably expect. The question I have for Thomists is whether they believe that all atoms have the same behaviors as those atoms observed by scientists validating their theories, or if they believe that some atoms - those in humans - behave in ways which do not conform to the mainstream physical theories.

    BenYachov,

    I found the paper; it's just called "Hylemorphic Dualism", which is nice and straightforward. I was thinking of the sentence followed by footnote [30]. He does seem to be - as you suggested - referring not to the things which are called "elementary particles" by physicists, but rather to hypothetical entities which are "spatially extended" but are "essentially elementary and hence indivisible", the possibility of which he denies. So, not denying the existence of (modern) atoms, but rather Democritus's Atoms. Good call.

    In that paper Oderberg does something which I'm coming to realize many other Thomists also do: he claims that the things which happen inside an organism - chemical reactions, for example - are determined by the macroscopic form of the organism... but he does not make it entirely clear whether he thinks that the atoms and molecules actually behave the same way when they are inside an organism as when they are outside of it.

    "The substantial form is what determines the permissible and impermissible behavior of the quarks in the body, which is why some chemical reactions typically occur, others rarely, and others not at all."

    Here, he strongly implies that the reason certain chemical reactions happen inside an organism is because of the form of that organism, rather than because of the properties of the molecules in the reaction. Should this be taken as a claim that the behaviors of the molecules inside an organism are actually different from the behaviors the same molecules would exhibit if they were in a similar immediate environment, but outside an organism? It certainly seems like it... but that's such a fundamentally strange and implausible claim! Does anyone here believe this?

    ReplyDelete
  95. Gyan, you posted while I was writing my latest post. I see that you understand the substance of my point; forgive me for re-iterating it to you above.

    ReplyDelete
  96. >Here, he strongly implies that the reason certain chemical reactions happen inside an organism is because of the form of that organism, rather than because of the properties of the molecules in the reaction.

    Cheers Steve glad I could help but if he doesn't explicitly say that then I am skeptical that is what he meant.

    Cheers again.

    ReplyDelete
  97. @Steve Ruble:

    "Here, he strongly implies that the reason certain chemical reactions happen inside an organism is because of the form of that organism, rather than because of the properties of the molecules in the reaction. Should this be taken as a claim that the behaviors of the molecules inside an organism are actually different from the behaviors the same molecules would exhibit if they were in a similar immediate environment, but outside an organism? It certainly seems like it... but that's such a fundamentally strange and implausible claim! Does anyone here believe this?"

    Correct on to the first statement (with some qualifications needed to prevent equivocations); the second is more difficult to gauge because I cannot tease out the exact meaning of "similar immediate environment".

    Take the simple example of an electron. A free electron behaves in a certain way while a bound electron in a hydrogen atom behaves in completely different way, a behavior that is delimited and dictated by the essence of the substance called hydrogen atom. In other words, because the electron is bounded in the hydrogen atom, it cannot be viewed as substance in and of itself, in isolation of the atom, but rather its own nature, which includes its own powers and essential properties as flowing from its essence, is harnessed or subsumed under the Form of the hydrogen atom of which they are part -- to use the Scholastic jargon, the electron is *virtually* in the atom, or the electron has no substantial identity of its own. There is nothing particularly mysterious in this (as Brandon remarks above) and the scientific a posteriori explanation for the case of the electron, can be readily found in any QM text, but the *metaphysical* point remains the same when substances, things that can exist in and of themselves, organize as parts of another substance.

    ReplyDelete
  98. @Gyan:

    "I don't think Thomists believe in the atomic hypothesis. To them, a living body is a substance in its own right and its nature, esp its final causes is not determined by its material constituents."

    Not an expert, not even a local one, but yes that is essentially correct. Substances are not reducible to their material constituents: take a cat. Now pulverize the cat in its material constitutes. On the one hand you have a cat, on the other you have a heap of atoms (or whatever units of matter you might wish to take as fundamental). The two are obviously different. What accounts for the difference? The essence of the cat. What is the essence of the cat? That is a matter for a posteriori investigation, from biologists, physicists, cat lovers, etc. The *metaphysical* point that Thomists make is that a cat, like other substances (this is a term of art) have an essence, the essential what-ness that makes the particularly lump of matter that makes up a cat, actually a cat and not a dog, a rock, etc.

    ReplyDelete
  99. Steve Ruble,

    Aage Bohr was quite serious about atoms, in the quantum mechanical sense, being purely theoretical constructs. See this letter in Physics Today.

    Aage Bohr, Ben R. Mottelson, Ole Ulfbeck, H. Dieter Zeh, and Michael Gerver
    Citation: Phys. Today 57(10), 15 (2004); doi: 10.1063/1.1825253
    View online: http://dx.doi.org/10.1063/1.1825253

    ReplyDelete
  100. grodrigues,

    By "similar immediate environment" I don't mean anything particularly complicated, just the idea that it's possible to conceive of an environment outside of an organism in which the particles which interact by way of the fundamental forces are present in roughly the same proportions and distributions as within an organism. For example, consider a cell in vitro as opposed to a cell in an organism: the particles within that cell are in a "similar immediate environment" to the particles within the cells within an organism.

    An bound electron within a hydrogen atom does not behave in a way "delimited and dictated by the essence of the substance called hydrogen atom", it behaves in a way delimited and dictated by the electromagnetic forces between the electron and a proton. We call such a system a "hydrogen atom", but it seems exactly backwards to describe the hydrogen atom as causing the behaviors of the electron and proton which compose it. Why do you think it's more helpful to describe the parts in terms of the system, rather than the system in terms of its parts?

    ReplyDelete
  101. Steve Ruble,

    If I may interject, you say "Why do you think it's more helpful to describe the parts in terms of the system, rather than the system in terms of its parts?".

    The issue here is that an object needs to be specified in terms of its constituents, i.e. its parts, but also must contain a structural component. To take a (perhaps overly) simple example, a certain ham sandwich cannot be said to exist simply when the sum of its ingredients exist - rather, there must be a certain structure of said ingredients before the ham sandwich can be said to exist.

    Now there are those which want to say that human beings per se don't really exist, rather what exists is a certain sum of elementary particles arranged human-wise. However, such a reductionist can't in principle say that such a sum arranged human-wise exists, without implicitly summing over just those micro-particles which exist where common sense supposes a human being to be (which is of course begging the question). For instance, a quark in someone's heart would have just as much causal interaction with a quark in someone's spectacles as it would with a quark in someone's glasses, or within the air molecules trapped within someone's pocket. There is nothing at the level of MP interactions which ties together just those particles which constitute the human being under analysis and rejects those which lie without it. Therefore, medium sized objects (such as human beings, lumps of gold, trees etc) are causally invisible at the level of MP interactions.

    However, clearly what we take to be medium sized objects do have causal properties and relations within the world, but these aren't reducible to MP interactions. Therefore, something else is required for the full picture of such objects, and this is a certain structure or form. There are other ways of coming at this point - but I believe this is the angle that Oderberg was taking. See Crawford J Elder's articles (which are freely available on his website) for further info.

    ReplyDelete
  102. @Steve Ruble:

    "An bound electron within a hydrogen atom does not behave in a way "delimited and dictated by the essence of the substance called hydrogen atom", it behaves in a way delimited and dictated by the electromagnetic forces between the electron and a proton. We call such a system a "hydrogen atom", but it seems exactly backwards to describe the hydrogen atom as causing the behaviors of the electron and proton which compose it."

    Andrew T already answered, much better than I ever could, but allow me to stress that your alleged correction is circular, for to say that the behavior of the hydrogen atom is dictated by the electromagnetic forces between an electron and a proton is just another way of saying that the essence of an hydrogen atom is, at least in part, to be constituted by an electron and a proton. And this is not enough, because one proton in one corner of the universe and an electron in another corner do not thereby constitute an hydrogen atom, a precise relation between the two must obtain. In the usual QM textbook treatment of the hydrogen atom, this is encapsulated by setting the potential term in Schroedinger's equation to the rotationally invariant Coulomb potential. To recognize that an electron and a proton alone do not thereby constitute an hydrogen atom but an accidental unity is tantamount to recognizing that there is something over and above in the ensemble of an electron and a proton that makes up an hydrogen atom. In other words, there is no non-circular scientific account of the hydrogen atom that does not refer, implicitly or explicitly to the whole. An hydrogen atom is one of the simplest in the hierarchy of natural substances, but as we crawl up higher and higher in the hierarchy more and more striking is the irreducibility of substances to their material constituents, until we reach man, which is by definition a rational animal, with the specific difference of rationality entailing that man cannot, not even in principle, be reduced to its physical parts even if we understand physical parts in a rather broad sense (I am not arguing this point, I am just stating it -- the OP provides one such argument).

    You clearly do not speak the language. When learning a new language there is always a tendency to translate everything in our known terms. While this is inevitable, it is also the cause of many pitfalls. As the proverbial saying goes, if all you have is a hammer, everything will look like a nail. The last quoted sentence "describe the hydrogen atom as causing the behaviors of the electron and proton which compose it" exhibits this, I would say. First, we are attempting a metaphysical description of hydrogen atoms, not a physical one. Confusing the two levels of explanation is just the scientistic non-sense in a different guise, without prejudice to the fact that questions about the essence of this or that substance are indeed informed, but not exhausted, by discoveries in physics (which by the way belies the alleged in-fallibilist position of essentialist metaphysics, although the charge, in the terms that it is usually put, is rather irrelevant anyway). Second, saying that the hydrogen atom causes the behaviors of the electron and proton, is prone to equivocation; rather, the substantial natures of both proton and electron are virtually in the hydrogen atom, their essential powers and attributes subsumed and harnessed under the whole substance, and thereby their behaviors are constrained and delimited by being in the hydrogen atom -- this is why the behavior of a free electron is different from the behavior of an electron bound in an hydrogen atom and different to that bound in say, some conducting material like copper wire.

    Similar analysis applies to your in-vitro cell. Being a substance of its own, its material constituents are subsumed to the in-vitro cell. But if the cell is part of a living body, then it too is subsumed under the Form of the living body and has no substantial identity of its own.

    Hope it helps.

    ReplyDelete
  103. You clearly do not speak the language.

    Ah. You're trying to teach us poor proles Newspeak -- the language in which it is impossible to think something that differs from the party line of the Party/Church.

    Now if only you can forget that Oldspeak still exists you can also cease to be troubled by the thought of a world outside of God's Dominion (i.e. your own strange, bigoted mind.)

    Orwell would be proud.

    ReplyDelete
  104. @Ray:

    "Ah. You're trying to teach us poor proles Newspeak -- the language in which it is impossible to think something that differs from the party line of the Party/Church."

    It is not every day that one has the displeasure to encounter such a consumate idiot as yourself. Saying that Steven Ruble does not speak the language and so is apt to equivocate has the exact same meaning as saying that someone who does not know the language of Mathematics is apt to equivocate its terms.

    ReplyDelete
  105. Cute stuff, Ray. Now anyone who doesn't agree with you must have a strange and bigoted mind. Just because the man stated that they are not in the same page and there is some terminology that needs to be understood and clarified (a perfectly reasonable recommendation, and something that scientists often do) then he's got to be an authoritarian bigot who is trying to tell everyone else what to do.

    Or maybe you're just mad.

    ReplyDelete
  106. Ah yes grodrigues. equivocation. Please teach us the difference between Hesperus and Phosphorus. Then we may also learn of the difference between your God and the voices inside your head.

    ReplyDelete
  107. anon

    disagreement with me does not necessarily make one a bigoted authoritarian. Excessive agreement with the man who wrote:

    "If this seems to be an angry book, that is because it is."

    That's another story. I myself am not angry though, just trying to fit in.

    ReplyDelete
  108. grodrigues,

    Well, I'm not going to contest your claim that any account of a thing is going to include a account of that thing, but I'm really having a hard time following you from there. I think you and I have wildly different concepts about what constitutes an "account" or an "explanation" for the way something is. It certainly does feel like we're speaking different languages.

    For example, you say:

    "First, we are attempting a metaphysical description of hydrogen atoms, not a physical one."

    On the other hand, Anonymous @2/27 5:02 wrote, "...metaphysical views (whether naturalism, materialism, hylemorphism, panpsychism or otherwise) don't get decided in the laboratory.", which is a sentiment I've seen echoed by other A-T folk around here (including Feser).

    I am completely at a loss to understand what it would mean to have a "description" of a physical object which cannot be be compared to any observable aspect of that object. What is it about a hydrogen atom that you intend to describe? What makes it the case that your "description" is actually "about" a hydrogen atom?

    "... the substantial natures of both proton and electron are virtually in the hydrogen atom, their essential powers and attributes subsumed and harnessed under the whole substance, and thereby their behaviors are constrained and delimited by being in the hydrogen atom..."

    You've added more words, but it still sounds to me like you're saying that the atom is causing the proton and electron to behave in particular ways. I'm really baffled as to what you're intending to claim if it's not that. But since the atom has absolutely no existence above and beyond the relationship that a proton and electron form when they have energy levels and proximity which allow them to bind to each other, I really don't know what you mean at all.

    ReplyDelete
  109. Andrew T,

    "Now there are those which want to say that human beings per se don't really exist, rather what exists is a certain sum of elementary particles arranged human-wise."

    One might as well say, "Sand piles don't really exist, rather what exists is sand arranged in various piles," amiright? OK, so the assertion that human beings "don't really exist" sounded silly until I remembered that for Thomists the phrase "really exist" means something different from what it means to most people.

    "However, such a reductionist can't in principle say that such a sum arranged human-wise exists, without implicitly summing over just those micro-particles which exist where common sense supposes a human being to be (which is of course begging the question)."

    I don't understand what question you think they are begging. Isn't there a rather radical difference between the following claims?

    A) Human beings "really exist" in a Thomistic sense, implying Forms and assorted metaphysical things.

    and

    B) Our common sense idea of human beings turns out to be strongly correlated in most respects with certain particular things our perceptual systems render to us as discrete entities.

    "There is nothing at the level of MP interactions which ties together just those particles which constitute the human being under analysis and rejects those which lie without it."

    I agree. But that seems more congenial to my point of view than to the Thomist's. I don't think that there are any differences between the MP interactions within and without a human being; I don't think there's anything "special" about the particles which are inside a human being as compared to those which are not.

    "However, clearly what we take to be medium sized objects do have causal properties and relations within the world, but these aren't reducible to MP interactions."

    Why not? Which interactions between medium sized objects could not - in principle - be examined at greater and greater levels of detail until the entirety of the interaction could be described in terms of a fantastically large number of MP interactions? Of course, we don't have the ability to do that, and are therefore forced to make much more approximate claims about the ways in which things interact. But I don't understand why you think we need to posit the real existence of "something else" in order to explain these interactions.

    ReplyDelete
  110. This comment has been removed by the author.

    ReplyDelete
  111. grodrigues,

    I just realized that I misunderstood what you were saying here:

    "Confusing the two levels of explanation is just the scientistic non-sense in a different guise, without prejudice to the fact that questions about the essence of this or that substance are indeed informed, but not exhausted, by discoveries in physics (which by the way belies the alleged in-fallibilist position of essentialist metaphysics, although the charge, in the terms that it is usually put, is rather irrelevant anyway)."

    Well, I don't know that I actually do understand what you're saying, but it seems like I was mistaken to accuse you of putting forward a description which was not tied to reality in some way. But I am still confused about what a metaphysical "explanation" is actually supposed to do, besides give you a deeply confusing way of referring to things. Is there any respect in which metaphysical assertions about the nature of things can be used to generate information about the physical nature of things, as opposed to the other way around?

    ReplyDelete
  112. @Steve Ruble:

    ""...metaphysical views (whether naturalism, materialism, hylemorphism, panpsychism or otherwise) don't get decided in the laboratory.", which is a sentiment I've seen echoed by other A-T folk around here (including Feser)."

    That a substance has an essence is a metaphysical question that cannot be decided in a laboratory; what the essence of a given substance is, say an hydrogen atom, is a question whose answer can be, is in fact, informed by whatever the lab inhabitants tell us. And for some kinds of essences (e.g. those requiring elaborate instruments and intricate measuring devices, like a particle accelerator), exclusively so, but in general it is the job of the informed metaphysican, or anyone capable of rational though really.

    "What is it about a hydrogen atom that you intend to describe?"

    Its essence, that is, that what makes an hydrogen atom an hydrogen atom. Why we need essences to account for substances or why all this talk cannot be cashed out in some other terms or reduced to scientific descriptions -- which is what I guess you want to know -- are questions best reserved for a book of (essentialist) metaphysics.

    If you think you can escape doing metaphysics, then think again. To quote, E. Burtt:

    Even the attempt to escape metaphysics is no sooner put in the form of a proposition than it is seen to involve highly significant metaphysical postulates. For this reason there is an exceedingly subtle and insidious danger in positivism. If you cannot avoid metaphysics, what kind of metaphysics are you likely to cherish when you sturdily suppose yourself to be free from the abomination? Of course it goes without saying that in this case your metaphysics will be held uncritically because it is unconscious; moreover, it will be passed on to others far more readily than your other notions inasmuch as it will be propagated by insinuation rather than by direct argument. . . . Now the history of mind reveals pretty clearly that the thinker who decries metaphysics . . . if he be a man engaged in any important inquiry, he must have a method, and he will be under a strong and constant temptation to make a metaphysics out of his method, that is, to suppose the universe ultimately of such a sort that his method must be appropriate and successful. . . . But inasmuch as the positivist mind has failed to school itself in careful metaphysical thinking, its ventures as such points will be apt to appear pitiful, inadequate, or even fantastic.

    "What makes it the case that your "description" is actually "about" a hydrogen atom?"

    I am not sure I understand your question; there is of course the possibility that we got the essence of a particular substance, say an hydrogen atom, wrong since grasping the essence of a substance is a combination of observation and intellectual judgment, but what conclusion do you want to draw from this epistemological fact?

    "I'm really baffled as to what you're intending to claim if it's not that."

    And I am baffled by your bafflement. At this point, I am afraid nothing that I could say would add anything so my suggestion is a simple RTM.

    edit: deleted previous comment to add Burtt quote.

    ReplyDelete
  113. "Even the attempt to escape metaphysics is no sooner put in the form of a proposition than it is seen to involve highly significant metaphysical postulates."

    The attempt to escape eating fat is impossible. Therefore, eat only fat. It's the only way to eat.

    ReplyDelete
  114. I'm new at this, so sorry for butting in, but for me an example that helps is qualia. It seems somewhat arbitrary (perhaps) to talk about how to look at a hydrogen atom, whether the whole determines the behavior of the parts or the parts determine the behavior of the whole. But on the level of consciousness, qualia such as "redness" certainly seem to be an example of something coming out of "nowhere," or not being reducible to the sum of its parts. It is easier to visualize this way for me. For atoms, molecules, photons, or light waves are not red, but when we perceive these things they are red. The form of "red light" seems to have properties that are not reducible in any way to its part (it does not seem coherent to talk about "components" of redness or more "basic" elements of redness).

    ReplyDelete
  115. Are there any limits to your metaphysics?

    Remind me how you know when you're right or wrong? What would tell you you are wrong? Intuition? A hunch?

    Where does the human begin and end? Which atoms? Food stuffs? Waste? When they are incorporated excreted? At what point in metabolism? What are the bare essentials to make a human? Is there a standard?

    Are drugs that change brain chemistry part of the human? What range of chemistry is allowed by your metaphysics?

    ReplyDelete
  116. @ Steve Ruble

    Here's what's going on in the mind of the Thomist wrt the hydrogen atom example (they will probably deny this because it's embarrassing)

    They take two statements about the electron and proton within a hydrogen atom

    1)they are found inside a hydrogen atom.
    2)rather than being described by just any solution of the QED equations for the electron-proton system, they are described by a bound-state solution. (otherwise it wouldn't be a hydrogen atom.)

    Rather than noticing that 1 and 2 are equivalent statements that describe the same thing, like a sane person, They do the following:

    1)They first observe that the description they thought of first caused the other one (because, you know, cause must precede effect.)

    2)They, then, conclude (stupidly) that because the first description caused the second description in their head, the thing described by the first description caused the thing described by the second description.

    hence, "essence of atom" causes the electron-proton system's wavefunction to occupy a restricted subspace of the electron-proton Hilbert space, and absolutely not vice versa.


    @ Syphax

    The natives will hate me for this, but read Dennett's "Quining Qualia" and see if you still feel the same way about Qualia when you're done. It's Googleable.

    ReplyDelete
  117. I've read Dennett, and probably feel quite a bit like "the natives" do on him.

    ReplyDelete
  118. Steve Ruble,

    Andrew T wasn't saying that he or Thomists were saying that "human beings per se don't really exist, rather what exists is a certain sum of elementary particles arranged human-wise"; he was saying that this is one of the standard philosophical positions one finds today. And this is right: it's one of the common ways of formulating a reductionistic materialism, a position Thomism opposes. Andrew T's point was that even formulating the position runs into problems. If you disagree with it as well, I think you're actually a lot closer to hylemorphism (although maybe not the Thomistic version of hylemorphism) in your position than you think.

    ReplyDelete
  119. @Syphax

    Not all that surprising, given that you know the word. I'm not sure if this describes you, but a lot of people seem to dislike his argument because it seems to deny more than it actually does. He's rejecting the bad explanations associated with the term "qualia" rather than the reality of what they are trying to explain -- which he calls "conscious experience" in his essay.

    Of course you may merely dislike Dennett because he is most unequivocally denying the God of the Nicene Creed, but that is another matter entirely.

    ReplyDelete
  120. Right, but something that has been defined as ineffable can only yield "bad explanations." That's the whole point - if qualia were easily expressible or transferable then they would pose no problem. He can argue that ineffable things can't figure into our arguments or decisions with one another, and I'd agree except that everything I seem to experience - everything - is composed of qualia. So the only question is, should I privilege the world of "effability" or the world of "ineffability" in how I live? Unfortunately I have no idea how to jump from the latter to the former - there's no good bridge to get there. All my trips there seem like short expeditions that must return at the end of the day. Why does Dennett demand that "real" things be explainable or quantifiable anyway? How could he even prove such a thing without begging the question? In any case, I feel quite confident in continuing to point out that red light has properties that individual light particles do not. Form does seem to impart something new.

    ReplyDelete
  121. And PS, I have never claimed to believe in a Nicene Creed. Not sure what that has to do with it.

    ReplyDelete
  122. @Syphax

    "He can argue that ineffable things can't figure into our arguments or decisions with one another, and I'd agree except that everything I seem to experience - everything - is composed of qualia."

    Hence the Wittgenstein quote:
    "Whereof one cannot speak, thereof one must be silent." (that's not the one quoted in Dennett's essay, but it speaks to the same point.)

    However, if qualia are not ineffable, perhaps those things that are imaged by MRI scans are qualia too, and they only seem different from your own qualia because your relation to them is different from your relation to your own qualia. i.e. other people's qualia can be visible to you, but they look different because they are perceived from a different perspective. You cannot see my qualia from my perspective because you are not me.

    (The MRI is not really quite the instrument we want. It only images blood flow, so many different sorts of qualia will look the same on an MRI scan.)

    red light has properties that individual light particles do not.

    Only in that it is seen as such by you. Light can't just appear red. It has to appear red to someone. At least that is both Dennett's view and my own.

    Form does seem to impart something new.

    It seems to. But does it?

    ReplyDelete
  123. @syphax

    On the Nicene creed, it was just a guess. I now see my mistake. Aristotle, not Aquinas.

    ReplyDelete
  124. I'm not a philosopher, I'm a graduate student of psychology (so I know a thing or two about MRIs), and I grew up in a (non-Nicene, some would even say non-Christian) religion that teaches materialism. As such I really have very few emotional hangups that cause me to prefer non-materialist theories of reality. I'm not even that committed to Aristotle - I just am not convinced by the materialism that everyone seems to want to force on me through bullying and sleight of hand (like redefinitions, in Dennett's case). The only thing I can think of is that materialism has psychological appeal - it's emotionally comforting to think that the universe should be quantifiable, tangible, measurable, and even deterministic because there is a comfort in knowing what to expect. Things that are immaterial and intangible present an existential challenge to those who want to be able to control and manipulate what is "real."

    ReplyDelete
  125. @syphax

    I'm a physicist by training, but a computer scientist by vocation. I guess being a physicist makes one accutely aware that in order to do anything useuful, you have to switch descriptions (or effective field theories) all the time, but none of the descriptions is the same thing as what it describes. You get in trouble when you try to give the "one correct description" of a thing. Because you forget that the description is not the thing it describes.

    What bothers me is not so much the claim that indescribable things exist, but the claim that one has described such a thing. Whatever it is you're talking about, it's not indescribable.

    As for quantification, it's sometimes a useful tool, because it provides precision, but it's no substitute for accuracy(economists often fall into this trap.)

    My feelings on determinism are mixed. My favored interpretation of quantum mechanics (MWI) is deterministic in a sense, but it isn't from the perspective of experience, (I don't know on which branch of the wavefunction I will find myself after it splits, whether both are real or not.) so I'm not sure what this really gets me in terms of control. I just think wavefunction collapse is an unneeded complication.

    The other thing that annoys me about anti-materialism is denying that scientific conclusions are possible, when in fact they are. Usually this takes the form of conveniently forgetting that Ockham's razor can, and must be, part of the scientific toolkit (and a theory with no exceptions is a lot simpler than one with a few non-negotiable but rather poorly documented exceptions. i.e. the miracles of the revealed religions.)

    Anyway. It's nice meeting you. Hopefully I don't seem like much of a bully (although this is a strange place to go if you don't want to be bullied -- Feser's style of rhetoric is, if anything, much worse in this regard than anything the New Atheists have writen, and in my experience it encourages his fans to do the same.) As for being unconvinced by materialism -- that's fine -- it's ok to be biased, as long as you have an open mind -- well, I'd prefer if you were a materialist/naturalist/physicalist/whatever you want to call it, because I think it's the correct position, but it's better to remain unconvinced than to become convinced for the wrong reasons.

    ReplyDelete
  126. I had a couple beers at my pub theology meeting this evening, so I won't try to say anything too complicated, but:

    Syphax,

    How seriously have you considered the fact that non-materialism has it's own psychological appeal? It's emotionally comforting to think of yourself as categorically different from mere animals, as having a mystical component, and as having a part which may live on after you die. The idea that dead is dead and that we are animals presents an existential challenge to those who want to be metaphysically special and live forever.

    Brandon, you may want to re-read my post. Perhaps it was unclear, but my position is approximately the opposite of what you seem to think it is.

    Ray, I agree with a lot of what you say, but I don't think our interlocutors here are concluding anything "stupidly". They're wrong about complicated things for complicated reasons, but I don't think stupidity is one of those reasons.

    ReplyDelete
  127. Steve,

    So in other words you think that human beings don't actually exist as such -- that the only things that exist are particles and that calling something a human being is simply a matter of convenience? I still don't see that in your comment; most of what you say suggests a particular form of nonreductive materialism, not a reductive one -- that you are claiming that the human being and a particular system of particles are equivalent, and so one can treat either of them as existing rather than having to say that only particles exist and everything else is projected. So you are a reductive materialist and do accept the claim that there are no human beings as such, only particles arranged humanwise?

    ReplyDelete
  128. rather than having to say that only particles exist and everything else is projected

    Sorry, that should say "rather than having to say that only particles exist and everything else is projected or a derivative theoretical construct".

    ReplyDelete
  129. Steve Ruble,

    "One might as well say, "Sand piles don't really exist, rather what exists is sand arranged in various piles," amiright? OK, so the assertion that human beings "don't really exist" sounded silly until I remembered that for Thomists the phrase "really exist" means something different from what it means to most people."

    Not sure what you're getting at here. I'm not advocated that "human beings don't really exist", rather I am saying that there are those who are of an ontological monist bent who want to say that all human beings really are, are certain elementary particles which are arranged human-wise. In other words, human beings don't really have an ontological status over and above what constitutes them.

    "I don't understand what question you think they are begging. Isn't there a rather radical difference between the following claims?

    A) Human beings "really exist" in a Thomistic sense, implying Forms and assorted metaphysical things.

    and

    B) Our common sense idea of human beings turns out to be strongly correlated in most respects with certain particular things our perceptual systems render to us as discrete entities."

    Those aren't the claims I was juxtaposing. The point is the reductionist must be able to describe said discrete entities using only the ontological components he or she recognises. However, there is no way of doing this at the level of MP physics, which you agree with further below. The only way to do so is take our common sense notions of objects, which the ontological monist must say are illusory, and perform a mereological sum in some way over these objects. Therefore in order to preserve medium-sized object's as causal entities in the world (which of course they are), he needs to first assume their existence while trying to build up a picture whereby all that truly exists are MPs - which is begging the question.

    "I agree. But that seems more congenial to my point of view than to the Thomist's. I don't think that there are any differences between the MP interactions within and without a human being; I don't think there's anything "special" about the particles which are inside a human being as compared to those which are not."

    The only way this is more congenial to your point is if you assume that there cannot be two way causation between the constituents and the structured whole. For instance, take a quark in a DNA molecule. This quark is constrained in its behaviour within the DNA molecule, and therefore a portion of its causal history will be determined by the macro-operations of said DNA molecule. However, at the level of MP physics, the DNA molecule is invisible (as you agree). Therefore, the DNA molecule cannot be solely reduced to operations at the MP level (though of course MPs and their interactions are constituents of said DNA molecule), yet the DNA molecule constrains its constituent MPs.

    ReplyDelete
  130. Steve Ruble,

    "Why not? Which interactions between medium sized objects could not - in principle - be examined at greater and greater levels of detail until the entirety of the interaction could be described in terms of a fantastically large number of MP interactions? Of course, we don't have the ability to do that, and are therefore forced to make much more approximate claims about the ways in which things interact. But I don't understand why you think we need to posit the real existence of "something else" in order to explain these interactions."

    Tell me how you would, in principle, describe the cause of a sudden movement of a quark in a glass molecule, which is part of a glass window, shattered by a baseball launched from an automatic baseball launching machine, solely by resorting to MP interactions (i.e. without taking into account medium-sized objects and their actions, i.e. the movement of a baseball and all the inter-related functions and components of the baseball launching machine)? How could you, in principle, specifically isolate causal series that results in that quark movement, which involves all and just those MPs which are involved in the medium-sized objects which common sense supposes are causal actors in the window breaking, without in sum way summing over those medium-sized objects?

    Note, it's no good just taking a "movie" of all the MPs and their interactions within a spatio-temporal segment, and "tracing" the chains which interact with said quark over the temporal period, for a couple of reasons. First, many other MPs will be involved in causal chains which interact with said quark over the temporal period, which do not feature in what we know to be the relevant causal series (i.e. the baseball hitting the window). However, it seems there will be no way of isolating just which causal chains cause the rapid movement of the quark without resorting to macro descriptions relating to the glass and baseball. A way of identifying a cause and effect pair is by demonstrating an invariance between the two - for instance, if I swing a cricket bat at a glass vase with greater or lesser intensity, there will be a greater or lesser intensity in the shattering of the vase. More formally, a variation in some property of a cause c, from p to p', should cause a corresponding and proportionate (given background information) variation of a property of the effect e, from g to g'.

    cont..

    ReplyDelete
  131. Now, how is the reductionist going to establish such an invariance relationship between, say, the speed of the MPs in the baseball and the speed of movement of the quark, without referring to the medium-sized objects? For instance, the reductionist could say that a group of MPs (which we would say are within the volume of the baseball) moving with greater or lesser speed towards a certain spatial region, will cause a greater or lesser speed of movement in the quark. Or, the reductionist could say that the group of MPs moving with greater or lesser speed towards another set of MPs of a certain type, will cause a greater or lesser speed of movement in the quark. However, setting up such counterfactuals is very problematic for the reductionist - due to the requirement of drawing out general causal descriptions. For instance in the first case, because the reductionist can't talk about the glass pane, rather only a spatial location in his or her causal description, it could be that there are simply no MPs of the right kind in that spatial location. Similarly, if the causal description only talks about the "baseball" MPs moving towards MPs of this or that kind, it may be that said "glass" MPs aren't in the right plane etc. In other words, under the above description, the cause event could be exactly replicated without the effect occurring at all - hence no invariance can be established. The reductionist can't talk about both the location and kind of MPs in the "glass" because he or she is then essentially summing over a medium-sized object.

    These are all difficulties for only the simple baseball + glass case. It becomes even more nightmarish for the reductionist in the baseball launcher + baseball + glass case. The hylemorphic dualist does not face such difficulties of course, as he or she can talk about both forms (i.e. baseballs, panes of glass and their macro-properties etc.) and their constituent matter (i.e. quarks, glass molecules, electrons etc.).

    ReplyDelete
  132. I don't deny that non-materialism has emotional appeal, it's just that this fact is pointed out so often that I figure it goes without saying. I will also agree that the rhetoric here is charged far beyond my tastes, though I don't Think it's worse than what I've seen on Coyne or PZ Meyers blog. It's unfortunate that this type of discussion drowns out meeker participants who might have much to say. You will see me around more.

    ReplyDelete
  133. @Syphax

    I will also agree that the rhetoric here is charged far beyond my tastes, though I don't Think it's worse than what I've seen on Coyne or PZ Meyers blog.

    I don't really buy this. For one thing, I can't quite imagine our current host posting something quite so useful as this colorfully worded piece of advice:

    http://scienceblogs.com/pharyngula/2006/08/a_suggestion_for_the_comments.php

    Furthermore, generally when PZ or Jerry produce charged rhetoric, it is in response to a greater provocation than mere disagreement. Feser also has an unbecoming habit of gloating whenever anyone credible says something vaguely similar to what he believes. This seems very much like a bullying move, since Feser is basically arguing that he's right because he has an army of scholars backing him up. It's also rather dishonest to downplay the differences between his own philosophy and those of the people he cites as making it more credible.

    I see Feser's fans as doing much the same thing: e.g. Jack "Vaughn" Bodie's comment of February 28, 2012 3:04 PM (on this thread) -- where he clearly implies that Arthur agrees with him (in disagreeing with me) far more than he actually does.

    In short, while PZ and Coyne may be just as mean as Feser at times, it's done much more in a spirit of fair play.

    ReplyDelete
  134. Syphax said:

    I will also agree that the rhetoric here is charged far beyond my tastes...It's unfortunate that this type of discussion drowns out meeker participants who might have much to say.

    This definitely is a problem. It would be improved, as far as the comments section goes, if we had more objectors like Steve Ruble above and fewer trolling commenters. Argument around here would tend to be fairly heated anyway, since the discussion here tends in practice, at least when trolls don't derail it, to be devoted to hammering out ramifications, refinements, and modifications of Thomistic ideas and argument; and since most of the regulars here have, for one reason or another (sometimes because they're Thomists and sometimes because they disagree with Thomism but find it interesting), an intense interest in and liking for finer details of Thomistic ideas and arguments, arguments get developed pretty boldly and discussion of particular details in such arguments can get pretty intense. We've always had a problem with commenters trying to crash the party rather than contribute to it, though, and that has led to fairly regular exasperasted impatience or cold amusement, which often takes it way beyond just some enthusiasts arguing about nuances, in much the way a chess club would get if there were people who kept coming to informal chess get-togethers to talk loudly about how stupid chess is, and how the rules don't make any sense, and how much smarter they are than any chessplayer because they wouldn't waste their time moving horsies and towers on a board at random and for no reason, etc....

    Admittedly, we probably could sometimes do better, despite this.

    ReplyDelete
  135. Ray,
    How would you justify that eternal branching of wavefunction is more sparing of Ockham's razor than the wavefunction collapse?

    ReplyDelete
  136. Ray,
    Why do you think that the hypothesis that thoughts are irreducible to atomic motions is not the most parsimonious today, given that no standard and acceptable physical theory of thoughts exist at present.

    Won't you agree that it is just materialist dogma to pretend that thoughts must be reducible to atomic motions? Really what empirical evidence you have?

    ReplyDelete
  137. Brandon,

    I'm sorry I didn't write what I meant in the first place; I forgot irony doesn't really work on the internet. I was trying to point out that there are other things in the world which we seem to be comfortable referring to as discrete entities ("sand piles") even though problems arise when we try to formally define what those things comprise (cf. the sorites paradox). See below...

    ReplyDelete
  138. Andrew T,

    Yea, I obviously failed to explain what I meant with the piles of sand comment, sorry, see above.

    I don't think there's a logical problem with claiming that the actual world is fundamentally a continuous and heterogeneous collection of particles while simultaneously acknowledging that we routinely consider certain sets of particles as discrete entities for purposes of reasoning about the world.

    Consider a scientist who is studying fluid dynamics. They might induce a vortex in a bowl of water and then study the ways in which water molecules enter and leave the vortex. In such a case, the water "in the vortex" is indistinguishable in essence from the water surrounding it, but the scientist can nevertheless pick out a fictional "entity" (the vortex itself) on which to focus by describing its limits or borders in a pragmatically useful way, and can make statements about that "entity". I think that this is analogous to what we do when we identify any entity as a discrete thing.

    I think there is a persistent issue in your extended remarks on the quarks in the ball-hitting-a-window scenario (which I found interesting and helpful, thanks). The issue is that you're not taking into account what a comprehensive description of a macro event in reductionist terms would actually encompass. If we had access to the facts about every particle in the scenario, we wouldn't any longer be talking about a ball and a window, we'd be talking about a bajillion particles (and it would take us a long time).

    "How could you, in principle, specifically isolate causal series that results in that quark movement, which involves all and just those MPs which are involved in the medium-sized objects which common sense supposes are causal actors in the window breaking, without in some way summing over those medium-sized objects?

    ... it seems there will be no way of isolating just which causal chains cause the rapid movement of the quark without resorting to macro descriptions relating to the glass and baseball."

    See? You're talking about two different levels of description at the same time. You're challenging the reductionist to explain a set of MP interactions delimited in terms of medium-sized objects, then objecting when the reductionist limits the MP interactions they are considering to only those which occur within the volumes you've picked out as areas of interest.

    The real question is: is there any MP in the volume of interest which experiences acceleration due to some thing other than its fundamental force interactions with its MP neighbors? If there is no such acceleration of any MP, it seems to me that all things happening within the volume of interest must in fact be explicable in terms of those MP interactions.

    ReplyDelete
  139. Ray not donald -- quick recap:

    ARTHUR (to Ray): I know I keep saying this, but I'll say it again. You're blowing smoke. You boast that you have "enough reason" to reject Thomism, but don't tell us what that reason is. All you can offer are appeals to its unpopularity. You boast about "unfounded premises", but can't name any.

    RAY: Here're some unfounded premises. At least one for each of the five ways: [...] I tried to only go for premises which were not just unfounded, but rather doubtful as well.

    ARTHUR: Granted, the important premises you've quoted may seem doubtful to you, but I'm sure a Thomist could defend them.

    RAY: Um. Arthur. You do realize that unfounded is a weaker claim than false (and in fact weaker than doubtful.) Right?

    ME: Odd that you should take that superior tone with Arthur when Arthur is, in fact, quite right: these are not unfounded premises but rest directly on primary principles of reason [...]

    (emphasis added)

    It seems to me you took on a tone of trying to belittle Arthur by implying he’d mistaken your weak claim for something stronger. He hadn’t. You did explicitly call out the supposedly unfounded premises as doubtful. Arthur expressed a belief (indeed he said he was “sure”) that a Thomist could defend them. I pointed out that he was not a deserving recipient of your snark as he was quite right: your quotes are not unfounded premises, and Thomists can defend them.

    In what way have I implied greater agreement with Arthur than was justified? Arthur, if I mischaracterized your words I apologize.

    grodrigues has twice pointed out that what you hold as true of “Feserites” (snark but no argument; coming “dangerously close to equating“ unfounded with doubtful) are failings you yourself exhibit. You can’t keep your own claims straight in your head. But now we’re the Orwellian Newspeakers apparently.

    Might it be prudent for you to read as directed and come back when the Two Minutes Hate is over?

    ReplyDelete
  140. Gyan

    How would you justify that eternal branching of wavefunction is more sparing of Ockham's razor than the wavefunction collapse?


    An eternally branching wavefunction (where each branch contains objects that may be approximately described using classical mechanics.) is nothing more than a description in (mostly) plain English of what solutions to the Schroedinger equation look like (at least those that follow the second law of thermodynamics.) The Schroedinger equation is pretty well forced on us by experiment; wavefunction collapse is not.

    If we declare that wavefunction collapse is real

    1) We have to guess when it happens -- and we're almost certain to be wrong.
    2) Even if we remain agnostic on the point of when wavefunction collapse is supposed to happen, it's still an extra element added to the theory. If we just start with the Schroedinger equation, we can already tell that the Copenhagen interpretation is what any observer (that can be described by a classical Turing machine) will see.

    Why do you think that the hypothesis that thoughts are irreducible to atomic motions is not the most parsimonious today, given that no standard and acceptable physical theory of thoughts exist at present.

    a reduction of one theory to the other simplifies the overall theory that encompasses both, because it reduces the number of free parameters. Number of free parameters is the standard of simplicity that is ACTUALLY USED in science (by way of calculating statistical significance of experimental results.)

    Of course you have to know roughly how to do the reduction. Otherwise, "choice of reduction" would be an extra free parameter. But, it's not like we don't have plenty of clues from neuroscience there. (Indeed, the very name of the field indicates that we know which cells have features we should be able to identify with mental states.)

    OTOH, suppose we assume it's impossible in principle to reduce mental states to the firings of neurons. Then we have a different free parameter which is just as problematic as choice of reduction -- choice of interaction. It's pretty clear that mental states interact with the physical world (since we talk about them by moving air particles around.) So we haven't actually helped ourselves by withholding judgment on whether reduction is possible in principle.

    Won't you agree that it is just materialist dogma to pretend that thoughts must be reducible to atomic motions?

    No. (Also, demanding that someone agree with you is most definitely a bullying move. I think it would be considered beyond the pale, even by us crude PZ Myers fans.)

    ReplyDelete
  141. In short, while PZ and Coyne may be just as mean as Feser at times, it's done much more in a spirit of fair play.

    This comment made my day.

    ReplyDelete
  142. @Ray:

    "Here's what's going on in the mind of the Thomist wrt the hydrogen atom example"

    First we got Ray, the Psychoanalyzer of Thomists, now we get Ray, the Mind Reader of Thomists.

    "hence, "essence of atom" causes the electron-proton system's wavefunction to occupy a restricted subspace of the electron-proton Hilbert space, and absolutely not vice versa."

    First, your language is consistently sloppy and inaccurate. A wave function does not "occupy a restricted subspace" of some state Hilbert space, but is a member of said subspace. This distinction is not mere pedantry, because wave functions are usually represented as functions -- more generally, sections of vector bundles -- on space-time, so they can be said to occupy regions of space-time. Second, you seem to think that the hydrogen atom state Hilbert space is a subspace of the state Hilbert space of the electron-proton system -- maybe because in some vague, unspecified way, the former is a subsystem of the latter -- but this is, strictly speaking, false. Third, what is the Hilbert space of the electron-proton system? Maybe you mean the tensor product of the Hilbert spaces for the electron and proton systems? But then what is the Hilbert space of an electron system? The Hilbert space of free electron? If you go with this you end up with the wrong answer. If not of the free electron, then what? And what about the proton? Since it is not a lepton but a 1/2-spin baryon composed of 2 up quarks and one down, do you want to bring in the full standard model to describe the hydrogen atom? Maybe some phenomenological simplification? Either way, guess what, that is not how the physical description of the hydrogen atom from first principles gets done.

    Lastly, and more importantly, I explicitly warned Steve Ruble against using equivocating language such as saying that the essence of the hydrogen atom caused the behaviors of the electron and proton -- you may be a mind reader, but your reading skills leave a lot to be desired.

    Your comments on physics, as well as your earlier comments on mathematics (yes, even your corrected description of Goedel's theorem is still muddled and inaccurate) are mediocre at best. You know nothing about (essentialist) metaphysics and Thomism, so what can you add to the discussion? Do you really have nothing better to do than trolling and showing-off your ignorance?

    ReplyDelete
  143. Steve,

    Thanks for the clarification; I'm still a little unclear on what kind of reductionism your view actually is, but rather than pester you with questions I'll see if it comes out in your discussion with Andrew T.

    As a side point, though, I'm myself not a reductionist about the sandpiles case; I don't think either account has any priority: my view is that they are conjugate accounts at exactly the same level of analysis, not a reducing and reducible account at different levels of analysis. In context it sort-of sounded like you had a very roughly analogous view but (unlike me) extended it to everything, and that was what probably what threw me off, more than the irony itself.

    ReplyDelete
  144. grodrigues

    1)The set of standard model states with baryon number 1 and lepton number 1 is a perfectly well defined Hilbert space. Yes there's an energy above which you can't necessarily describe it as a proton-electron state (but it's much higher than the ground state energy plus the binding energy of the atom) and I suppose there could also be any number of neutrino anti-neutrino pairs in there (but they are very weakly interacting, so it would take a long time for them to matter.) And of course there will be an electromagnetic field (possibly containing one or more photons) which I believe I mentioned in my original post. In any event, I don't see how mentioning these details adds to the discussion. I suspect the majority of people on this blog neither understand nor care about these details.

    2)Fine. I already admitted that I was phrasing Godel's theorem wrong, but since you will continue to harass me until I get it right:

    There are true statements that are not provable given the following constraints on the meanings of the terms "true" "statements" and "provable"

    a)The claim, that a certain set of equations involving multiplication and addition over the natural numbers, has a solution is a statement.
    b)such a statement is either true or false, but not both. (This seems like an absolute must if you want to be any sort of a mathematical realist.)
    c)provability can be verified by a Turing machine. (This is true of all known proof systems over a finite number of axioms.)
    d)All the other things generally assumed about the above terms in math papers hold (natural numbers follow the Peano axioms, provable statements are true, etc., etc.)

    You happy now?

    I strongly suspect that accusations of equivocation on your part are merely a defense mechanism. i.e. if I draw a conclusion from what you said, that sounds embarrassing to you, it clearly can't be what you meant. If, however, I am wrong and I continue to be ignorant about essentialist metaphysics after your many posts, what does that say about your skill as a communicator?

    ReplyDelete
  145. @Ray:

    "I suspect the majority of people on this blog neither understand nor care about these details."

    These details are only important insofar as they show your previous claim about "absolutely not vice versa" to be unproved, nothing more, nothing less.

    "You happy now?"

    Not really, but lest I be misunderstood for a stalker I am going to let you off the hook.

    "I strongly suspect that accusations of equivocation on your part are merely a defense mechanism."

    You over and over again psychologize the "natives" and "Feserites" here. Whatever your reasons for doing so, it is nothing but an ad hominem attack and a sure sign of trollish behavior.

    "If, however, I am wrong and I continue to be ignorant about essentialist metaphysics after your many posts, what does that say about your skill as a communicator?"

    Independently of my skills as a communicator, whether good or bad, my "many posts" in this thread on the issue, with a couple of exceptions, only touched a single point that Steve Ruble brought up in his reading of "Hylemorphic Dualism". Do you really expect your ignorance to be dispelled in a combox? Am I supposed to take this expectation honestly? If you persist in your ignorance is because you choose to as references have already been given.

    ReplyDelete
  146. As someone watching this conversation, I can't help but notice that Ray is absolutely wrapped up in pretty typical internet atheist wharblegarble. Everyone who disagrees with him is, he insists, insane and crazy. Only he and everyone who agrees with him has grasped the truth. And when others disagree with him or point out the problems with his positions, it's all some kind of evil trick, or mind games, or he feels bullied.

    The icing on the cake was him saying that Feser is a bully, because he points out people who agree with him. But Coyne and Myers aren't bullies, because they feel provoked (ie, someone disagreed with them) most of the time they start ranting, and their namecalling is totally okay.

    And since Ray loves to tell everyone what they're thinking, I'll go ahead and say Ray comes off as someone who isn't used to being disagreed with (he's used to an echo chamber) and being swatted around intellectually without cheerleaders backing him up is leading to some panic on his part. Some cognitive dissonance, where the fact that he's doing pretty poorly in an argument has to be blamed on conspiracy, or people being mean, or... anything, but his own shortcomings, or (worse) the shortcomings of his philosophy, or (worst of all) the shortcomings of the movement he supports.

    My two cents, since psychologically analyzing people is a move he keeps on with. :)

    ReplyDelete
  147. Anonymous troll

    Way to circle the wagons there. The fact that a single hostile poster can inspire such terror on a conservative Catholic website, hosted in a country where conservative Catholics are a majority/or near majority(depending on how you count Kennedy) in the highest judiciary body and 2 out of the top 3 republican candidates for president are conservative Catholics, speaks volumes about the weakness of the intellectual position.

    ReplyDelete
  148. This South African wants to know if USA is really a predominately Catholic country? I was under the impression that Catholics were a minority in the US.

    I don't think the conservative Catholic position is weak. It's just that being a troll is very easy. For example we know how easy it is to troll evolutionary sciences. Spreading FUD is one of the easiest things in the world.

    ReplyDelete
  149. "The fact that a single hostile poster can inspire such terror on a conservative Catholic website... speaks volumes about the weakness of the intellectual position. "

    The trouble with this is that you'd be better off looking at Catholic arguments if you want to discover their weaknesses, not the emotions of individual Catholics. I've pointed this out at least once, but you still don't seem to get it. In the arena of reason, arguments are what matter. Focus on them. You consistently use psychoanalysis in place of analysing arguments, and you still don't seem to understand that it's irrelevant. What matters are arguments. Surely as a fellow follower of reason, you can see that?

    ReplyDelete
  150. Anonymous

    This South African wants to know if USA is really a predominately Catholic country? I was under the impression that Catholics were a minority in the US.

    The US is 75% Christian and about 25% Catholic. That said, the protestants are of many denominations, so Catholic is the largest single denomination in the US. See http://en.wikipedia.org/wiki/Religion_in_the_United_States for more detailed figures.

    That said, Protestants can loosely be grouped into evangelical (mostly white, conservative, and more anti-intellectual than the Catholics), mainline (liberal to moderate Christians), and historically black (self explanatory, hopefully. They tend to be socially conservative and politically liberal.)

    The evangelical block is larger than the Catholic block, and more ideologically unified, however an interesting political dynamic has developed over recent years with respect to the supreme court. Republican presidents almost always want to nominate a religious conservative, but pretty much everyone who's not an evangelical thinks the evangelicals are crazy. The Catholic Church has conservative leadership (although US Catholics are no more conservative than the country as a whole) and is less overtly anti-intellectual than evangelical churches. Thus, All 5 Republican nominees to the Supreme court have been Catholic. One of the four Democratic nominees is Catholic as well (although she is more representative of the community of immigrants from Latin America, who are politically liberal and socially moderate.)

    Also, two of the major candidates in the Republican Presidential Primary this year are conservative Catholics (Santorum is a true believer, Gingrich is a bit of a phony from what I can tell.) The frontrunner is a Mormon, though.

    (I think the above description is largely accurate, but I'm not going to pretend I can hide my biases.)

    In summary, Catholics have a LOT of power in the US. MUCH more than atheists. Even in academia, there are a many Catholic run Universities http://en.wikipedia.org/wiki/List_of_Roman_Catholic_universities_and_colleges_in_the_United_States and zero universities run by a constitutionally atheist organization. Point being, if it seems like Catholics are losing the intellectual battle, it is not because society has an institutional bias against them.

    Also, I would note that academics tend to be the least susceptible to FUD campaigns, at least in my experience. So it is curious that academia is precisely where Catholicism, and religious positions in general have lost the most ground.

    Anyway, hopefully this explains my point a little better and is at least fairly informative about US politics and demographics.

    ReplyDelete
  151. I think Ray is committing the all-too-common mistake of confusing politics and psychology with philosophy. On and on he goes about precicely how much power Catholics have, or how afraid they are of him, or whether they seem to be losing the "intellectual battle". No wonder he was so ready to appeal to popularity; he can't see the difference between politics and rational argument. Either that, or he honestly thinks that political victory matters more than reason.

    You've cleary got a lot to say, Ray, but you still can't focus on the arguments, and it's all too easy to see why; you don't know anything about them.

    ReplyDelete
  152. Steve Ruble was mature enough to accept my little correction about Atoms, atomic units of matter and Democritus.

    That tells me he really wants to understand the arguments & philosophies that compete with his materialist views. If only so he can be a better defender of materialism and more credible critic of our views.

    That is my kind of Atheist/Agnostic/skeptic. The rational non-Cult of the Gnu type.

    Notice that accepting that correction on his part is not the same as conceding "Gee you guys must be right about this Thomism stuff and materialism must be wrong".

    Gnus like Ray OTOH aren't thinkers. They feel Theism is wrong and like some Pentacostals I've met, no amount of logic will pierce their divinely inspired feelings.

    ReplyDelete
  153. Arthur and BenYachov

    I already stated which premises I disagree with in Aquinas' arguments as well as Popper/Searle's. Rather than arguing for those premises, you and your fellow posters asked me to do their work for them. This is a stalling tactic, not an argument.

    The only difference between me and Steve is that I have decided that since you are only trying to argue that your position is respectable, rather than that it is right, I will deny you the respect you so desperately crave. I think this is justified, given how cowardly it is to stop seeking the truth and settle for respect instead.

    ReplyDelete
  154. Also,
    I find it ironic that Arthur's post accusing me of playing politics was followed by BenYachov's nakedly political attempt to compliment Steve by tearing me down. (Divide and conquer. Eh?)

    ReplyDelete
  155. Brandon,

    "As a side point, though, I'm myself not a reductionist about the sandpiles case; I don't think either account has any priority: my view is that they are conjugate accounts at exactly the same level of analysis, not a reducing and reducible account at different levels of analysis. In context it sort-of sounded like you had a very roughly analogous view but (unlike me) extended it to everything, and that was what probably what threw me off, more than the irony itself."

    Well, when I say that a sand pile "exists", I just mean that I think there are a bunch of sand grains arranged together in a particular way; likewise, if I say that a human being "exists" I mean that the matter in a particular volume of space is arranged in a such way that it contains something I call a human being. So there's a sense in which I think that, yes, of course, sand piles and human beings exist. But what I mean seems to be different from what Thomists mean when they say that human beings "exist": they seem to want to say that there's something else that makes a certain arrangement of particles be a human being. I don't believe in that "something else", because a) I don't know what it is, and b) I don't see how it could work.

    ReplyDelete
  156. @Ray:

    "I already stated which premises I disagree with in Aquinas' arguments as well as Popper/Searle's. Rather than arguing for those premises, you and your fellow posters asked me to do their work for them."

    It is a marvel to behold, and it really never ceases to amaze, how clueless some people are. You say that you have already *stated* your disagreements and then ask us to *argue* for the premises and do the work for you. Amazing.

    Ok, let me play by your rules: I disagree with you, so argue your case. And no asking me to do the work for you.

    "The only difference between me and Steve is that I have decided that since you are only trying to argue that your position is respectable, rather than that it is right, I will deny you the respect you so desperately crave."

    Given how many times you have repeated a thought similar to this one, this may come as a shock, but you are not *that* important that your "respect" merits any craving.

    ReplyDelete
  157. >I find it ironic that Arthur's post accusing me of playing politics was followed by BenYachov's nakedly political attempt to compliment Steve by tearing me down. (Divide and conquer. Eh?)

    So every Atheist should automatically be on your side in everything because they are Atheists?

    That is political.

    Speaking for myself the fact someone like let's say George R. is a Theist doesn't automatically mean I am on his side.

    (In fact 8 out of 10 time I am not)

    So enough of the boring political crap.

    Let me be blunt.

    Learn some f***ing back round philosophy or f*** off. You are wasting everybody's time. I am enjoying from the sidelines the intelligent exchange between Steve and Brandon.

    You have nothing to contribute. You are like a Young earth Creationsit with a 5th grader's knowledge of biology trying to pick a fight with 3rd year college biology major.

    We have to spend more time correcting and educating you than answering you.

    Do your own homework.

    ReplyDelete
  158. @BenYachov

    If you don't want me to respond to your posts, all you have to do is ignore me (i.e. not gratuitously insult me in your posts.) Why is that so hard for you to do? Surely no one would believe a word an ignoramus like me said, whether you pointed out my ignorance or not.

    ReplyDelete
  159. Heck I'm not the blog master here you can do what you want Ray & Dr. F's good pleasure.

    Ok stay & with your continued asinine posts prove to all of us Gnu'Atheists are no better than brain dead fundies except
    without belief in gods.

    We do seem to have an opening for a Gnu'Atheist clownbot since Dr F kicked djindra out of here.

    No doubt you will fill that role nicely till you become similarly boring.

    ReplyDelete
  160. Hay wait why didn't I think of that before?

    Obsession with politics? Can't be corrected? Acts like he knwos it all?

    Ray=djindra?

    Ya think?

    ReplyDelete
  161. Oh. And, can't be corrected?

    I admitted my misstatement about Godel's theorem immediately (perhaps too soon, as my original statement was better than my second attempt, in that it was only imprecise, not wrong.) And yes, I am also aware that I left off one assumption from my third attempt (the claim that a statement is provable is itself a statement.) Oh well, I'm not perfect. But I most certainly do admit mistakes.

    ReplyDelete
  162. Then prove me wrong Ray.

    Learn do your own homework or F*** off.

    Because I have as much civility towards Gnus as PZ Myers has for YEC anti-evolutionists.

    BTW I don't begrudge Myers' contempt toward Fundie YEC and their willful ignorance of biology and science.

    I begrudge his brain dead Gnu dogma that all religion is no different than fundie YEC. Coyne too.

    Because only a F***ing idiot can believe that.

    ReplyDelete
  163. On second thought, I don't think I actually need the extra assumption. Anyway, if one of you folks actually wants to be helpful, perhaps you can set me straight on that issue. Grodrigues seemed unhappy with the way I stated the theorem still, so I figured he had a good reason.

    ReplyDelete
  164. Better.

    Wither grodriguez wants to answer you is up too him.

    ReplyDelete
  165. BenYachov

    All religions are the same?

    Not so much. They all make different (and often mutually contradictory) false claims. Heck, some Unitarians, Buddhists, and Taoists might not even believe anything particularly unreasonable (It's rather hard to say what they believe.)

    But the claim that Aquinas's proofs establish anything beyond a reasonable doubt (let alone the existence of a mind, not confined to the skull of a human) is patently absurd.

    There's a reason people keep bringing up the same tired objections -- they still have not been soundly refuted. Sure you can move the problems around introducing "accidentally ordered causal series" and "essentially ordered series" and "active power" and "passive power." And you can give the floating assumptions of Thomist-Aristotelian metaphysics fancy names like the "principle of sufficient reason" (which has several versions, at least one of which is true but vacuous) and "principle of proportionate causality," (which, though weasel worded using phrases like "in some sense" probably can't be saved.) But in the end you still have not proven these principles from anything remotely resembling uncontroversial assumptions. In other words, a reasonable person may freely doubt or even reject all of Aquinas's conclusions. All Aquinas can say in his defense is that "to suppose otherwise would be absurd." A proof from personal incredulity if I've ever seen one.

    The best thing that can be said of Thomism is that it accurately captures the structure of human thought. But to claim it accurately depicts the fundamental structure of physical reality, or indeed something even more basic than that is to confuse map and territory.

    Now you can go on claiming I haven't proven any of the above statements. That's fine, I'm not the one who claims to have proven something to a degree of metaphysical certitude.

    ReplyDelete
  166. "There's a reason people keep bringing up the same tired objections -- they still have not been soundly refuted."

    Heh. No. Plenty of the "same tired objections" are borne out of misunderstanding and ignorance. People repeatedly argue "maybe the universe is eternal!" to argue against Aquinas' argument for a First Cause. It's a tired objection. It also happens to be one expressly borne out of ignorance of Aquinas: assume the universe is eternal, and it does nothing to his first cause argument.

    Which is by and large what we're seeing with you. But at this point you've backed off to "people can always reject your first principles, thus I say you haven't established the truth of your claims with utter infallible certainty". How far you've fallen in this thread. Coyne and Myers would grumble at how badly you've performed.

    I'd say you shouldn't care what they think, but I think we both know that would fall on deaf ears. Or blind eyes. ;)

    ReplyDelete
  167. Ray's last post must be seen as utterly inane. His 'argument', apart from repeatedly trying to dismiss Thomistic assumptions and position with extremely silly and ignorant one-liners, is basically his unargued and unsupported personal incredulity about the ability of discursive reasoning to reach firm conclusions in this area.

    This actually appears to have been his ' argument' all along. Ray, how about you actually try and formulate your critique of metaphysical reasoning into something like a coherent argument. Only then does it seem like we have to take your position with any kind of seriousness.

    As the principles of causality and proportionate causation are basic to our thought, and eminently rational, to the point that it is to going to absurd lengths to challenge them in order to attack Thomism, we are owed a lot more in terms of an argument against them than the dismissive and silly comments in Ray's last comment.

    Can you give what alluded Hume; a coherent argument for how the effects can be out of proportion to the cause or how anything that is contingent can have no cause?

    ReplyDelete
  168. Anon

    People repeatedly argue "maybe the universe is eternal!" to argue against Aquinas' argument for a First Cause.

    1) It's a fine objection to the Kalam argument, which is also frequently brought up by Theists and is therefore not a straw-man. You're not the only faction pushing some variant of the first cause argument you know. Historical confusion does not make a person's other views worthless.

    2) The real objection is that there's no reason to exclude infinite regresses a priori. While certain types of infinite regresses (in particular the ones where each level is understood in terms of the previous level) cannot exist in the mind, as minds are finite, there is no particular reason to doubt infinite regresses can exist in nature. (and since nature doesn't have to understand her elements in the same way as man, or indeed at all, the accidental/essential ordering distinction is of no help.) So like I said before, you've just moved the problem, not eliminated it.

    But at this point you've backed off to "people can always reject your first principles, thus I say you haven't established the truth of your claims with utter infallible certainty".

    Enh. Once you're out of your "it's a metaphysical statement so it's 100% absolutely certain" trap, there are plenty of evidential arguments that can knock down the God hypothesis as an unnecessary, and therefore probably false, complication, including the one in the God Delusion.

    And don't tell me God's simple -- oh really: monophysite or chalcedonian, nicene or arian, indeed Jewish, muslim, christian, zoroastrian etc. etc. Seems like a lot of free parameters to me. Not to mention, we have learned a thing or two about our own minds, and they're as complicated as all get out, so if God is supposed to be some sort of a mind, well -- say hello to William of Ockham's little friend.

    ReplyDelete
  169. "The best thing that can be said of Thomism is that it accurately captures the structure of human thought. But to claim it accurately depicts the fundamental structure of physical reality, or indeed something even more basic than that is to confuse map and territory."

    This. I wonder if anyone has written a paper contrasting folk accounts of causality and substance with the Thomistic accounts. Of course, you would need to do a lot of work translating the Thomistic terms into everyday language, but I bet you could find some very interesting parallels.

    ReplyDelete
  170. Westcountryman

    As the principles of causality and proportionate causation are basic to our thought,

    Perhaps your thought, but not mine. (Although I doubt it, since you didn't even know which two principles I was talking about.)

    "The principle of causality i.e. causes precede effects" can be seen as a consequence of the second law of thermodynamics, and I in no way recommended trashing it for the purposes of understanding the post-big-bang observable universe.

    The principle of sufficient reason in its most basic form is utterly vacuous, since "X is true" is always a sufficient condition for X. There may be more complicated sufficent conditions, which do not explicitly declare the truth of X to be an axiom, but "X is true" always suffices. It's the more complicated version with act and potency that gets you in trouble, and you're stuck trying to explain why God can have active potency, but nothing else can. (Aquinas uses the argument from degree here, which is the sort of thing that was already seen as suspect by Plato, (read "Parmenides"), when he pioneered that kind of argument.

    The principle of proportionate causation as stated by Feser contains the phrase "in some way." How could something so vaguely worded possibly be basic to thought? (The vague wording is probably because attempts to formalize the principle yielded conclusions that were obviously false.)

    In any event, the premises ACTUALLY used in mathematics and the natural sciences will do just fine thank you, and they don't seem to be leading members of the NAS to God in droves these days, now do they.

    Only then does it seem like we have to take your position with any kind of seriousness.

    positions like mine are already taken very seriously by far greater minds than the likes of you. I think it is you Thomists who have the bigger problem in this regard. I suggest you folks try harder rather than asking everyone else to do the homework you haven't done yourselves.

    ReplyDelete
  171. "It's a fine objection to the Kalam argument, which is also frequently brought up by Theists and is therefore not a straw-man."

    No, you ignoramus, it is a straw-man when you think this is an objection to the First Cause as argued for by Aquinas. Which is exactly what I said. It illustrates that you have no idea just what's being discussed or claimed.

    When a person says "evolution isn't true because I never saw a monkey give birth to a man", yes, if they thought they were arguing against evolutionary theory as Darwin conceived of it, their input is worthless. It doesn't become valuable just because someone out there advances hopeful monster theory.

    The fact that you can't even admit this much just shows how little of value your input is here. You're stuck in the mode where any criticism of Thomism has value, and any defense of Thomism has none, because darn it, you just hate anything theistic and that's that. It may soothe your psyche. Pity it's irrational, eh?

    "The real objection is that there's no reason to exclude infinite regresses a priori"

    Yes, there are reasons. As have been argued both in the context of Thomism as well as Kalam. That you are willing to accept them no matter what because otherwise "*screech* scary theism!" doesn't impress anyone outside of your crazy little internet circle.

    "Enh. Once you're out of your "it's a metaphysical statement so it's 100% absolutely certain" trap, there are plenty of evidential arguments that can knock down the God hypothesis as an unnecessary,"

    Nah, not at all. And again, there's your horrible reasoning on display. You think that, so long as you can just imagine some alternative to God, the alternative is absolutely 100% better because, again, "*screech* Scary theism!" Classical theism in general, not just thomism in particular, is vastly more preferable by weight of argument and evidence than materialism and/or atheism. The fact that you're trying to appeal to just the logical possibility of error speaks volumes here.

    As for TGD, Dawkins is a known nitwit when it comes to these topics. He has no knockdown arguments that can take down anything that isn't a strawman, and most atheist philosophers (and yes, even scientists) are well aware of that fact. The fact that he was made into a bitch on TV recently, after pussing out against WLC, just drives the point home. You need a better hero to worship, because the one you have has been exposed as a hack.

    The theists and non-materialists have the better arguments, better evidence, and the advantage of reason generally. Quick, put your fingers in your ears and hum a lot to make the Scary Truth go away. ;)

    ReplyDelete
  172. "This. I wonder if anyone has written a paper contrasting folk accounts of causality and substance with the Thomistic accounts. Of course, you would need to do a lot of work translating the Thomistic terms into everyday language, but I bet you could find some very interesting parallels."

    ROFL

    ReplyDelete
  173. "positions like mine are already taken very seriously by far greater minds than the likes of you."

    I think you meant far greater minds than yours, Ray. But why should anyone be impressed that your position is taken seriously by people of average intelligence? ;)

    ReplyDelete
  174. @Ray:

    Just stop, it is embarrassing. You do not know what you are talking about and are just making a fool of yourself. Proof?

    ""The principle of causality i.e. causes precede effects" can be seen as a consequence of the second law of thermodynamics, and I in no way recommended trashing it for the purposes of understanding the post-big-bang observable universe."

    First, you completely botched the principle of causality, second, no metaphysical principle is a consequence of a physical law -- this is in the "not even wrong" category.

    "The principle of sufficient reason in its most basic form is utterly vacuous, since "X is true" is always a sufficient condition for X. There may be more complicated sufficent conditions, which do not explicitly declare the truth of X to be an axiom, but "X is true" always suffices."

    There are various versions of the PSR, but none that I can recognize in your jumble of sentences, much less the Thomistic PSR, which is a metaphysical principle of being not an epistemological statement.

    "It's the more complicated version with act and potency that gets you in trouble, and you're stuck trying to explain why God can have active potency, but nothing else can."

    Maybe you mean that only God can be purely actual? That is actually easy to prove.

    "Aquinas uses the argument from degree here, which is the sort of thing that was already seen as suspect by Plato"

    Uh, I think you are deeply confused here.

    "The principle of proportionate causation as stated by Feser contains the phrase "in some way." How could something so vaguely worded possibly be basic to thought?"

    First, read up on what it means for a cause to contain what it imparts either eminently or formally. Second, are you suggesting that if some principle is not expressible in some formal theory, then it cannot be basic to our thought? Warning: think before you answer, lest you trap yourself in incongruity. Or do not answer, because most probably we will be treated to another display of your ignorance.

    "In any event, the premises ACTUALLY used in mathematics and the natural sciences will do just fine thank you"

    Fine for what? For doing physics? That is a reasonable position but it is false, because the *actual* practice of physics comes with a lot of unwritten metaphysical assumptions: there is an external mind-independent reality, our cognitive senses are reliable, there are other minds other than ourselves, etc. So, how do you justify them, and many others like these, if neither physics nor mathematics can do it for you? This is just a rhetorical question, I am not expecting you to answer, or even any desire to slog through another "answer" of yours.

    "positions like mine are already taken very seriously by far greater minds than the likes of you."

    That may or may no be so; it is also irrelevant. What is relevant is that you cannot rationally justify your position, not even if your life depended on it.

    Really, just sit down and read a decent book on the subject. Or just wallow in your smug, self-satisfied ignorance. Or whatever. But here in this blog, you are just playing the role of the village idiot, a fit target for derision and insult.

    ReplyDelete
  175. Ray, I agree with grodrigues that you are embarrassing yourself.

    I know you mentioned Sufficient Reason, but I took you to actually mean the two principles I referred to because I couldn't understand why you would be quibbling over Sufficient Reason. Indeed I still cannot, you are all over the place, totally out of your depth, and extremely hard to understand.

    Also as a Platonic Christian, and general lover of Plato, your garbling of The Parmenides is not appreciated, by the way.

    Anyway, as grodrigues has pointed out you misunderstand the 'principle of causality' in quite significant ways. The principle means that anything contingent has a cause. Now in one sense this is true by definition, because that is what a contingent thing is, but also it is true we cannot even conceive of something coming into existence without a cause, even in the realm of quantum mechanics where the natural quantum potentials in any particular situation amount to a cause.

    The same is true for the 'principle of proportionate causality'. If I had 5 gallons (unfortunately where I come from the 'godless metric system' has taken over, so I'm not sure if this is a reasonable or outlandish amount!) of cola for a party and I pour it out in my guests cups I would expect, all things being equal, the original amounts in their cups (before they had drunk any) to add up to 5 gallons. This is basic to our thought and reason. If it did not add up to this amount, but added up to more I would expect there would a reason, an external, efficient cause, for this, such as my guests adding spirits to their drinks.

    This is true no matter how we explain the exact relationship between the cause and the effect (whether for instance in a more Platonic or more Peripatetic way), undermining your silly point about the vagueness involved in the explanation of this relationship.


    positions like mine are already taken very seriously by far greater minds than the likes of you.

    Aside from the hilarity of your comment I think I'll point out my perspective, if you wish to get into this sort of discussion;

    Unlike professor Feser I have very little respect for 'modern philosophy'. Indeed one of my criticisms of his otherwise fine work is he takes modern and 'contemporary philosophy' too seriously. I agree with the wise man who once said that if Plotinus (or Plato or Aquinas etc,.) is a philosopher then Descartes (or Kant or Hegel etc,.) is not and vice vers;, and to me the former is a philosopher. By the time you get to Bertrand Russell and Ayers and their ilk my respect is, if possible, even less. And need I even mention the serial sophists of today like Dennett, who are in no sense philosophers? So your talk of 'greater minds' means little to me.

    ReplyDelete
  176. Steve Ruble,

    Thanks for the discussion so far – it has been interesting and nicely civil.

    “I don't think there's a logical problem with claiming that the actual world is fundamentally a continuous and heterogeneous collection of particles while simultaneously acknowledging that we routinely consider certain sets of particles as discrete entities for purposes of reasoning about the world. “

    I see this position as being a nominalist, conventionalist position : namely that the world is, as has been said before, a “play of properties” with no essential “joints” demarcating different objects from one another. Rather, what we consider objects are in fact our own projections onto the world, and these projections are jointly shared via convention and education. In other words, our conventions are logically prior to our assigning the status of objects or discrete entities to our experiences of the world.

    However, there is a consistency problem with this. Presumably, the conventionalist will presumably consider mental activity either equal to, or in some sense dependent on, brain activity. Therefore, the development, use and propagation of conventions are dependent on brain activity. But surely, brains are discrete entities which have some essential characteristics if they are to operate as brains – it is difficult to see how a conventionalist could deny this. For instance, a brain cannot operate if it is vaporised, or if it is squashed to fill a square inch container. So, on the one hand, conventionalists want to say that conventions are logically prior to our determination of discrete entities, but on the other, they also need to put the existence of brains (i.e. a distinct entity with essential properties) logically prior to conventions. Therefore, I don’t think this position is tenable.

    “Consider a scientist who is studying fluid dynamics. They might induce a vortex in a bowl of water and then study the ways in which water molecules enter and leave the vortex. In such a case, the water "in the vortex" is indistinguishable in essence from the water surrounding it, but the scientist can nevertheless pick out a fictional "entity" (the vortex itself) on which to focus by describing its limits or borders in a pragmatically useful way, and can make statements about that "entity". I think that this is analogous to what we do when we identify any entity as a discrete thing.”

    I disagree, the vortex is not simply a fictional entity unless you assume conventionalism. It is certainly a structure of form existing within the body of water and has properties distinct from the body of water in which it is formed. For instance, it has persistence conditions which are different to the body of water (i.e. the body of water will exist before and after the vortex) and therefore has properties which are distinct from the water. By Leibniz’s Law, the two can therefore be considered numerically distinct.

    ReplyDelete
  177. cont.


    “See? You're talking about two different levels of description at the same time. You're challenging the reductionist to explain a set of MP interactions delimited in terms of medium-sized objects, then objecting when the reductionist limits the MP interactions they are considering to only those which occur within the volumes you've picked out as areas of interest. “

    I’m trying to make a fairly subtle point here, and have no doubt failed to convey it properly. My point is that undoubtedly the relevant causal series between the quark movement and the baseball being launched involves just those MPs which constitute the baseball, the pane of glass and finally the quark. We know that, because we can set up invariance relationships fairly easy in such a causal series, using medium-sized object notions. The problem for the reductionist is to set up such a causal series (with invariance relationships) without explicitly or implicitly first assuming the existence of medium-sized objects. I’m not denying that if the reductionist was successful, that the causal series would involve just those MPs composing what we take to be medium sized objects. Rather, what I am denying is the ability, in principle (I agree, the computational task would likely be extremely lengthy and complex) of the reductionist to isolate said causal series without implicitly or explicitly using our supposedly fictional medium sized object boundaries and properties in their construction. Given that we know all the relevant physical laws, the failure of the reductionist to be able to proffer such an explanation must say something against the reductionist account and suggests that something else is required in the explanatory and ontological mix, i.e. forms. My argument above suggests that in order to establish a cause and effect relationship, one must be able to show an invariance relationship between the cause and effect. I gave reasons above why I think the reductionist account is unable to show such an invariance without appealing to the supposedly fictional medium-sized objects.

    ReplyDelete
  178. cont.


    ”The real question is: is there any MP in the volume of interest which experiences acceleration due to some thing other than its fundamental force interactions with its MP neighbors? If there is no such acceleration of any MP, it seems to me that all things happening within the volume of interest must in fact be explicable in terms of those MP interactions.”

    I don’t think your second sentence follows from your first; perhaps you could try to flesh it out some more? Again, the idea is a subtle one. When we talk of forms being in some way the cause of this or that, we are not speaking of anything that is going to result in violations of the law of conservation of energy. Think of it this way. In the assembling of a house from the relevant materials, energy is always conserved and in the process all MP movements no doubt are due to the local movements of other MPs – however, there has been a structuring of the materials into the form of a house which has real causal consequences.

    Now returning to the baseball and glass example. Surely, any description of the cause and effect series in this case must include some sort of information regarding the structure of the glass, which gives it its specific properties including its strength to withstand certain projectiles. Now you would probably argue that such information is contained within the propagation of the MP interactions, and you would be right – this isn’t surprising because the MP interactions are a sub-set of the interactions of the structures or forms of the launcher, the baseball and the glass (at least according to the hylemorphic account). In other words, just because when we rewind the “movie” we can see all of the MP interactions progressing “through” the relevant structures of the medium-sized objects, doesn’t mean that said structures or forms are reducible to the MP interactions (I take the ontological “realness” of the medium sized objects to be predicated on the unifying concept of form).
    This would be like making a “movie” of the house being constructed, noticing that when rewinding all one can see is the various interactions between the parts of the house, and therefore concluding that the house can be completely explained by simply the parts and their interactions. I feel that the deficiency of this account can be drawn out by first observing that the physics relating to the interactions between the parts of the house cannot be used to delineate the house from other spare bits of wood leaning on the side of the house, or water in the gutters from the recent shower. It can be drawn out in a second way by the failure to setup an invariance relation between the cause of the series (i.e. the builder’s action working according to a plan) and the effect (i.e. the completion of the house at some time t) without in some way implicitly or explicitly invoking the unity of the medium sized objects involved (i.e. builder and the house). The same goes of course for the baseball and glass example.

    ReplyDelete
  179. Steve Ruble,

    Just one other comment which might help illuminate what I am trying to get at above. Let’s make the (very big) assumption that all the current stable of elementary particles are truly that, elementary and not further reducible into parts. Now, all these particles have their set ways of interacting with each other, governed by the standard model. However, none of these laws of interaction are sufficient or specific enough to, say, build up a hydrocarbon molecule. Certain elementary particles have the potential to constitute such a molecule, but it’s not as if they have inbuilt “code” that would somehow enable them to self-assemble into one (in other, Thomistic terms, they need something to actualise that potential). Rather there is a certain “function” that the universe needs to execute “on behalf” of the said elementary particles in order for them to constitute a hydrocarbon molecule – in other words, they need to be transformed “by the universe” into hydrogen and carbon atoms, and then by the combination of certain pressures they can be transformed into the molecule. However, there is nothing in the elementary particles themselves that have a “formula” for a hydrocarbon molecule. In this sense then, the hydrocarbon molecule cannot be reduced simply to the elementary particles that constitute it, rather it needs a formal component along with the material component i.e. the elementary particles. In one sense, the formal component can be said to be immaterial, because it is a kind of function that “the universe” has imprinted onto the elementary particles (i.e. the matter) but is not contained there-in. However, the formal component isn’t in some way instantiated off in some Platonic realm, rather it only makes sense when it is united with matter. As you can see, there is nothing “magical” about this that somehow is additional to the forces between MPs and violates the conservation of energy.

    This form is a unifying principle of its constituents, which as a result constrains their behaviour while they are part of the object/form-matter composite. As such, you can’t really say that all causation is only efficient causation (i.e. interactions between MPs), because the formal component constrains the behaviour of the efficient causation. Because the formal component “extends” across the form-matter composite, the composite (or as I have been saying above, medium sized object) can be treated as a unified whole and is therefore, in ontological terms, a real object. Unified wholes are real causal actors, which I have been trying to outline above, and to follow Alexander’s Dictum (to exist is to have causal powers), therefore medium-sized objects/form-matter composites are, in all ontological strictness, real objects.
    Sorry for the mountain of text.

    ReplyDelete
  180. Westcountryman

    The principle means that anything contingent has a cause.

    Uck. If that's what you meant, you're worse off than I thought. The concept of "contingency" has no place whatsoever in any respectable discipline (well, that's not quite true, modal logic is a perfectly respectable mathematical formalism, but treating the possible worlds in any kind of realist sense runs into all the usual problems -- Russell's paradox, Godel's theorem etc. And Aquinas makes the further mistake of equating possible worlds with times in our own.) The closest thing people use when actually trying to figure something out about what's real is the concept of probability, which is treated in a realist way in some interpretations of Quantum Mechanics. Maybe purely actual is supposed to mean "existing in eigenstate," of some observable. But in that case we have to ask which one. If it's an energy Eigenstate, Parmenides (the philosopher, not the dialogue) would be right that purely actual things can't change (or interact with other things for that matter - when we describe an interacting part of a system as an energy eigenstate, we're strictly being imprecise.) In any event there's no law that any part of a quantum system has to be in an energy eigenstate.

    In summary
    1)Contingency is just a label we slap on elements of mental models (which we may consider applying to the world, depending on various unknown elements of our thought process or sensory inputs.) the model, and the labels we stick on it, are in our mind.
    2)Recognizing that it's useful to distinguish between the various real numbers between zero and one, it's better to use probability (in the Bayesian sense) for this purpose.
    3)Any principle making reference to the concept of contingency is only of specialized interest (modal logic), and therefore, not basic to thought.

    The same is true for the 'principle of proportionate causality'. If I had 5 gallons (unfortunately where I come from the 'godless metric system' has taken over, so I'm not sure if this is a reasonable or outlandish amount!) of cola for a party and I pour it out in my guests cups I would expect, all things being equal, the original amounts in their cups (before they had drunk any) to add up to 5 gallons.

    Oh, you mean conservation of mass -- while strictly not true, physicists do already assume that in some situations. If you mean the version that's actually true (conservation of energy,) it's of no help, because as far as we can tell, the total energy of the universe (including gravitational potential energy) is zero. Even if it wasn't zero, I don't think it would help, since you'd have to show that energy conservation was actually violated somewhere (quantum fluctuations don't count here.) Physicists also have found a few other conservation laws, but they don't seem to help your case either.

    Your form of argument seems to be
    1) Ancient philosophers didn't write clearly (Aquinas is no exception here), but I learned one specific interpretation which is 100% absolutely right.
    2) If someone misinterprets what I'm talking about when I say something, (perhaps because, you know, Christians don't use the terms consistently either,) it must be because they are ignorant.
    3) Refuse to commit to a specific interpretation of Aquinas before asking someone else to refute it.

    ReplyDelete
  181. Unlike professor Feser I have very little respect for 'modern philosophy'.

    Well then, you must not think that philosophy and the natural sciences are distinct disciplines (because this is a strictly modern distinction.) Do you also have no respect for modern physics, chemistry, biology, and mathematics, because those fields are also dominated by Atheists at the highest levels (in numbers, not some sinister political sense -- the only people that ask whether you believe in God at job interviews are theologians and their ilk.)

    ReplyDelete
  182. (well, that's not quite true, modal logic is a perfectly respectable mathematical formalism, but treating the possible worlds in any kind of realist sense runs into all the usual problems -- Russell's paradox, Godel's theorem etc. And Aquinas makes the further mistake of equating possible worlds with times in our own.)

    What are you blabbering about? The concept of "possible world" was introduced by Leibniz, modal logic was developed during the 20th century and modal realism is a recent and controversial view attributed to David Lewis. How the heck was Aquinas supposed to know about those things?

    And what do Russell's paradox and Gödel's theorem have to do with that? I'm starting to think that you're just trolling everyone here.

    ReplyDelete
  183. Andrew T,

    I'm enjoying this too.

    I think your synopsis of my position as "nominalist, conventionalist" is accurate, although I don't know enough about the technical baggage of those terms to tell whether I actually agree with them in general. I agree with this: "In other words, our conventions are logically prior to our assigning the status of objects or discrete entities to our experiences of the world."

    However, I feel like the consistency problem you propose has a problem of its own:

    "Presumably, the conventionalist will presumably consider mental activity either equal to, or in some sense dependent on, brain activity. Therefore, the development, use and propagation of conventions are dependent on brain activity. But surely, brains are discrete entities which have some essential characteristics if they are to operate as brains – it is difficult to see how a conventionalist could deny this."

    On my position, brains will "operate as brains" no matter how they are described. Our convention of differentiating the matter within the skull from the skull itself is something we do in order to pick out and talk about that stuff, but the only thing that a brain needs to operate as a brain is to be made of the right things put together in the right way so that it does what we call "operating". I don't think it needs some extra essence or "actual" distinctness to make it work.

    "So, on the one hand, conventionalists want to say that conventions are logically prior to our determination of discrete entities, but on the other, they also need to put the existence of brains (i.e. a distinct entity with essential properties) logically prior to conventions."

    Insofar as it's correct that our conventions are generated by minds that run on a substrate of brain, I guess it is correct to say that the existence of brains is logically prior to the existence of conventions. But what does not seem to be logically prior is the idea (or convention) of the brain as a distinct entity. I think the matter that composes brains will act the same way no matter how it is conceptualized.

    "I disagree, the vortex is not simply a fictional entity unless you assume conventionalism. It is certainly a structure of form existing within the body of water and has properties distinct from the body of water in which it is formed."

    I'm curious about whether Thomists have written much about vortices. It seems like it might be difficult to pin down what the form of a vortex is, and how you can tell whether some water has the form of a vortex or not. Something of a sorites paradox, but with a homogeneous, fixed amount of water molecules rather than sand or hairs.

    "My point is that undoubtedly the relevant causal series between the quark movement and the baseball being launched involves just those MPs which constitute the baseball, the pane of glass and finally the quark."

    What about the air molecules the baseball needs to push aside as it moves towards the window? It's probably not critical here, but I'm curious about how a Thomist would describe the interactions between the ambient air and the baseball, especially if there is a breeze pushing the ball aside from the window. Surely if anything can be accurately described in terms of the motions of molecules, it's the wind; and it simultaneously seems like something which would be quite difficult to describe as having a form.

    continued...

    ReplyDelete
  184. Andrew T,

    "We know that, because we can set up invariance relationships fairly easy in such a causal series, using medium-sized object notions. The problem for the reductionist is to set up such a causal series (with invariance relationships) without explicitly or implicitly first assuming the existence of medium-sized objects."

    Let's simplify this way, way down, so that we can get right at this invariance relationship idea. Suppose we have a perfect diamond cube 1mm on a side. We have a diamond piston which will propel the free diamond for some distance down a metal chute. So we have a cause c, the piston, with a property p, the force it applies, and an effect e, the distance the diamond travels, which will take a value g. So here we have a simple system. The behavior of all the atoms in the volumes we call "the cube" and "the piston" can be described in reductionist terms: they are all and only carbon atoms and they form an almost incompressible lattice in which force applied to one atom will be passed on to the next atom(s) along that vector.

    In such a situation, it seems quite plausible to me that we can set up an "invariance relationship" between the force p of the piston and the distance g traveled by the cube when struck by the piston: if the force is increased to p', the atoms of the cube will all receive an increased impulse from the piston and distance will increase to g' in a predictable way. The whole thing can even be described in an object agnostic way: "if the atoms in cylindrical volume X each have velocity v, after time t the atoms previously in cubical volume Y will be found in cubical volume Z; however, if the atoms in X have velocity v' then the atoms of Y will be found in volume Z'" or something like that.

    So, in my stripped down scenario, which causal series or object histories do you think are inexplicable by a reductionist? And which pertinent aspects of your baseball/launcher/window scenario have been left out?

    ReplyDelete
  185. Steve Ruble,

    I'm coming into this really late and I'm no expert in science or metaphysics but...

    I have a question for you about the baseball: Is a baseball completely explainable as a collection of elementary particles?

    What about the design of the baseball? Is that also explainable via a collection of elementary particles? I think not.

    At some point, it seems to me, there has to be some significance given to thought and mind in respect to the baseball. The design of the baseball determines much of the arrangement of its constituent parts.

    This is a parallel to how the Fifth Way applies to nature IMO. Constituent parts act differently depending on the whole they constitute.

    Take your in vitro cell example... The cell has the same parts as a cell in an organism but it behaves differently. The cell inside the organism acts as if it has a job to do. The in vitro cell just sits there until it dies.

    Aquinas said, quite brilliantly, (paraphrasing here) that things without minds act as if they have intentions. Like the baseball whose constituent parts artificially come together, according to a plan, to make a whole - so too does most every observable thing in nature act as if there is some plan or point or goal to which it must comply. In spite of the fact that there is really nothing different about the constituent parts in and of themselves.

    That's my simple, and quite late, take on it anyway. Great discussion so far!

    ReplyDelete
  186. Andrew T,

    I don't think I understood what you were saying with the house examples, because I was unsure what role - if any - the fact that houses are built by people (rather than coming together by themselves) was supposed to play in the account.

    "I feel that the deficiency of this account can be drawn out by first observing that the physics relating to the interactions between the parts of the house cannot be used to delineate the house from other spare bits of wood leaning on the side of the house, or water in the gutters from the recent shower."

    Well, hmm. The delineation between "spare bits of wood" and "house" qua "spare bits of wood" and "house" is something you are drawing; in reality, there are just a very large number of atoms which all have their own properties. But on the other hand, "physics relating to the interactions between the parts of the house" must also be in play in the fact that the spare bits of wood are not connected to the rest of the house; if you kick the spare bits away, the house doesn't move.

    I'm afraid I'm also not following your discussion of the hydrocarbon molecule; I don't know what you mean here:

    "Rather there is a certain “function” that the universe needs to execute “on behalf” of the said elementary particles in order for them to constitute a hydrocarbon molecule – in other words, they need to be transformed “by the universe” into hydrogen and carbon atoms, and then by the combination of certain pressures they can be transformed into the molecule."

    ...and I don't think I understand the rest of that post either. In seems like part of what you're saying is that a thing must be structured in the way that things which are that kind of thing are structured if it is to be that kind of thing, but that seems so obvious that there must be something more that you mean. (Incidentally, I can see why Thomists have constructed their own arcane vocabulary, if the alternative was to write sentences like that one.)

    ReplyDelete
  187. Anon

    How the heck was Aquinas supposed to know about those things?

    Exactly my point. If Aquinas had known about such things, he would have stated his terms more precisely (e.g. by either by stating outright that he was a modal realist -- and not just any modal realist, but one who believes that all possible worlds are realized at some time, or he would have realized that his assumptions were only applicable to the inside of his own mind.) I don't blame Aquinas for not noticing this. I blame his modern followers, who should know better.

    By the way, the reason Descartes is identified by Feser as the point when philosophy went off the rails is because he is the first Western philosopher to correctly identify the "necessary being" that is called God by Thomists -- "I think therefore I am". The "Last Superstion" Feser refers to is, therefore, the belief in a world outside of ones own mind (which cannot be absolutely proven -- solipsism is entirely self consistent. -- but one would be unreasonable to doubt it.)

    ReplyDelete
  188. Like the baseball whose constituent parts artificially come together, according to a plan, to make a whole - so too does most every observable thing in nature act as if there is some plan or point or goal to which it must comply. In spite of the fact that there is really nothing different about the constituent parts in and of themselves.


    That's a real muddled pair of sentences there! I'll try again (and hopefully not make it worse!)

    Like the baseball whose constituent parts come together artificially, according to a plan, so too do constituent parts in nature come together naturally as if according to a plan - in spite of the fact that there is nothing different in and of the constituent parts themselves.

    I don't know if that's any better or not. It IS shorter though!

    ReplyDelete
  189. As a clarification:

    "he was a modal realist -- and not just any modal realist, but one who believes that all possible worlds are externally realized at some time, or he would have internally realized that his assumptions were only applicable to the inside of his own mind."

    ReplyDelete
  190. Daniel

    When you ask whether something is explainable, do you mean

    1)an explanation exists in the mathematical sense (i.e. in the sense that the first prime number larger than a googleplex exists)

    or

    2)it is physically plausible that an explanation will be stated at some point in the future.
    ?

    The general rule is that a system (whether the universe or the universe plus God) cannot contain an unambiguous description of itself. (This principle is variously formalized by Godel's theorem, Tarski's theorem, and the halting problem. The details are tricky, as I observed up-thread.) Anyway, the fact that there is no highest Turing Degree: http://en.wikipedia.org/wiki/Turing_degree leads me to suspect that the concept of God does not solve the problem. The easiest way out is to just say that explainable and real are different concepts. This creates no contradictions as far as we can tell, but a 100% certain proof of this fact would either contradict Godel's second theorem or the fact itself.

    ReplyDelete
  191. @Ray wrote:

    >"It's the more complicated version with act and potency that gets you in trouble, and you're stuck trying to explain why God can have active potency, but nothing else can."

    God has potency? Since when?

    Ray you are an idiot and in spite of my best efforts you have nothing to contribute.

    Saying this in any class on Thomism gets you an F!

    ReplyDelete
  192. "And Aquinas makes the further mistake of equating possible worlds with times in our own."

    ?????

    Ray, c'mon man.

    you're really embarrassing yourself. seriously.

    ReplyDelete
  193. Of course you can consistently claim that there is a being, more powerful than any of Turing's oracle machines, however, it requires us to forget everything we ordinarily mean by saying that a being has "power." So every representation of the "being" is something static, unchanging, and dead.

    ReplyDelete
  194. d and BenYachov

    You are both making the foolish mistake of assuming that Aquinas never contradicted himself. This is the same thing that Biblical literalists do, trying desperately to find an interpretation of their sacred texts that is not an embarrassment, rather than assuming the author means what he says.

    ReplyDelete
  195. oh and BenYachov

    please explain the difference between "active power" and "active potency," and do it clearly.

    ReplyDelete
  196. @Ray

    >Oh, you mean conservation of mass -- while strictly not true, physicists do already assume that in some situations.

    Ray you are not just an idiot you are a f***ing idiot.

    You clearly can't comprehend the difference between a metaphysical modelling of nature vs scientific description in terms of physics.

    Aristotle erroneously thought for a rock to move from point A to B required an active force to continuously actualize it's movement otherwise it would stop. This is proven incorrect by Newton. Since an object in momentum stays in momentum till acted upon.

    The former & the later are scientific descriptions of physics(with Aristotle's being erroneous). Not metaphysical ones.

    But the basic description of the movement itself as change is what metaphysical modelling is all about. Parmenides denies the change is real(regardless of which Physics is correct). Heraclidus said only the change is real.

    Both doubt realism and the validity of out senses. Aristotle said both the change and permanence is real. Which he explained in terms of actuality and potency.

    This modelling is valid regardless of the physics involved(which are merely descriptions of mechanical process not of being). You can only be argued against metaphysical descriptions with philosophically not science.

    If you don't get this brute fact then you are wasting our time .

    But if you wish to still play the village idiot, the Atheist equivalent of the Young Earth Creationist rube who rants "The Second Law of thermal dynamics refutes Evolution" hey don't let me stop you.

    So go study or go F*** off.

    ReplyDelete
  197. BenYachov,

    I know the difference between physics and metaphysics. Metaphysics is the study of possible worlds (i.e. those that exist in one's own mind.) Physics is the study of the actual, real, external world. The claim that metaphysics is more general than physics, rather than merely seeming to be is exactly what's under dispute here.

    speaking of "active power," which I'm pretty sure is the same as "active potency" in the latin, here's your link. http://www.newadvent.org/summa/1025.htm . Next time you try to correct me, try to actually be right.

    ReplyDelete
  198. @Ray
    >please explain the difference between "active power" and "active potency," and do it clearly.

    No, do you own homework. Stop being such a lazy dick.

    >You are both making the foolish mistake of assuming that Aquinas never contradicted himself.

    I would never make that mistake. After all every faithful Catholic knows Aquinas got the Immaculate Conception wrong. But you clearly don't know enough to find a legitimate contradiction.

    >This is the same thing that Biblical literalists do, trying desperately to find an interpretation of their sacred texts that is not an embarrassment, rather than assuming the author means what he says.

    The Protestant Biblical literalists (who ironically not all that literal where it counts John 6 anyone?) assume the Bible is perspicuous and can be read privately apart from Tradition(2 Thes 2:15) and Church (1 Tim 3:15).

    We don't make those assumptions thus they are non-starter objections.

    Like I said you hold a one size fits all view of religion. They are all the same to the likes of you Gnu.

    I say we have given this jerkoff enough chances.

    Let's ignore him. He has proven beyond a shadow of a doubt he is a troll.

    ReplyDelete
  199. >I know the difference between physics and metaphysics. Metaphysics is the study of possible worlds (i.e. those that exist in one's own mind.) Physics is the study of the actual, real, external world.

    Wrong. Not even close.

    Good Bye.

    ReplyDelete